You are on page 1of 75

Online Prelims TEST - 25 (TEXTBOOK)

( InsightsIAS Mock Test Series for UPSC Preliminary Exam 2020 )

1 According to the Constitution, Goods and Services Tax (GST) Council will be a joint forum for the
Centre and the States. Consider the following about its composition.
1. The Union Finance Minister is the Chairperson.
2. The Union Commerce Minister is the Vice-Chairperson.
3. The Minister in charge of finance or taxation or any other Minister nominated by each State
government are members of the council.

Select the correct answer using the codes below.


A. 1 and 3 only
B. 1, 2 and 3
C. 1 only
D. 2 and 3 only

Your Answer : A
Correct Answer : A

Answer Justification :

Justification: The Goods and Services Tax (GST) is governed by the GST Council. Article 279 (1) of
the amended Indian Constitution states that the GST Council has to be constituted by the President
within 60 days of the commencement of the Article 279A. According to the article, GST Council will
be a joint forum for the Centre and the States. It consists of the following members: The Union
Finance Minister will be the Chairperson. Member – the Union Minister of State in charge of
Revenue or Finance. The Minister in charge of finance or taxation or any other Minister nominated
by each State government, as members

The GST council is the key decision-making body that will take all important decisions regarding the
GST. The GST Council dictates tax rate, tax exemption, the due date of forms, tax laws, and tax
deadlines, keeping in mind special rates and provisions for some states. The predominant
responsibility of the GST Council is to ensure to have one uniform tax rate for goods and services
across the nation.

Q Source: In news frequently

2 Consider the following statements.


The Hambantota port, sometimes seen in news
1. is a part of China’s Belt and Road Initiative
2. is strategically located in Maldives in the Indian Ocean

Select the correct answer using the codes below.


A. 1 only
B. 2 only
C. Both 1 and 2
D. only

Your Answer : A
Correct Answer : A

prelims.insightsonindia.com 1
Online Prelims TEST - 25 (TEXTBOOK)
( InsightsIAS Mock Test Series for UPSC Preliminary Exam 2020 )

Answer Justification :

Justification: Hambantota is right in the middle of vital energy supply lines in the Indian Ocean,
connecting the Middle East and East Asia Hambantota is the main town in Hambantota District,
Southern Province, Sri Lanka. This underdeveloped area was hit hard by the 2004 Indian Ocean
tsunami and is underwent a number of major development projects including the construction of a
new sea port and international airport.

Why is India worried?

India’s apprehensions about the apparently growing Chinese presence in the island are well known,
given the two countries’ competing strategic interests in the island. The Hambantota port is part of
China’s Belt and Road Initiative. Chinese control of Hambantota, which is part of its modern-day
“Silk Route” across Asia and beyond, as well as a plan to acquire 15,000 acres (23 sq miles) to
develop an industrial zone next door, had raised fears that it could also be used for Chinese naval
vessels

Q Source: Sometimes seen in news w.r.t OBOR

3 The least number of judges in the Constitution bench of the Supreme Court is

A. Two
B. Three
C. Five
D. Seven

Your Answer : B
Correct Answer : C

Answer Justification :

Justification: Constitution benches of India’s Supreme Court have delivered landmark judgments,
especially in recent months.

Constitution benches normally have five judges, but there have been benches with seven, nine and
even 13 judges.

Article 145(3) says at least five judges need to hear cases that involve “a substantial question of law
as to the interpretation” of the Constitution, or any reference under Article 143, which deals with
the power of the President of India to consult the Supreme Court.

Q Source:
https://www.insightsonindia.com/wp-content/uploads/2019/04/InsightsonIndia-Mar-2019-Current-Aff
airs.pdf

4 Consider the following statements.


1. A Free Trade Agreement (FTA) differs mainly from a Preferential Trade Agreement (PTA) in that
duties are reduced to zero on chosen items in FTA, whereas they are only partially reduced in a
prelims.insightsonindia.com 2
Online Prelims TEST - 25 (TEXTBOOK)
( InsightsIAS Mock Test Series for UPSC Preliminary Exam 2020 )

PTA.
2. A Comprehensive Economic Partnership Agreement (CEPA) is broader that a traditional Free Trade
Agreement (FTA) in that it also looks at the finer regulatory aspects of trade that FTA may not
cover.

Select the correct answer using the codes below.


A. 1 only
B. 2 only
C. Both 1 and 2
D. None of the above

Your Answer : C
Correct Answer : C

Answer Justification :

Justification: Statement 1: In FTAs, tariffs on items covering substantial bilateral trade are
eliminated between the partner countries; however each maintains individual tariff structure for
non-members. India Sri Lanka FTA is an example. The key difference between an FTA and a PTA is
that while in a PTA there is a positive list of products on which duty is to be reduced; in an FTA
there is a negative list on which duty is not reduced or eliminated. Thus, compared to a PTA, FTAs
are generally more ambitious in coverage of tariff lines (products) on which duty is to be reduced.

Statement 2: A Comprehensive Economic Cooperation Agreement (CECA) or a Comprehensive


Economic Partnership Agreement (CEPA) is different from a traditional Free Trade Agreement
(FTA) on two counts.

Firstly, CECA/CEPA are more comprehensive and ambitious that an FTA in terms of coverage of
areas and the type of commitments. While a traditional FTA focuses mainly on goods; a CECA/CEPA
is more ambitious in terms of a holistic coverage of many areas like services, investment,
competition, government procurement, disputes etc.

Secondly, CECA/CEPA looks deeper at the regulatory aspects of trade than an FTA. It is on account
of this that it encompasses mutual recognition agreements (MRAs) that covers the regulatory
regimes of the partners. An MRA recognises different regulatory regimes of partners on the
presumption that they achieve the same end objectives.

Q Source:
https://commerce.gov.in/writereaddata/trade/FAQ_on_FTA_9April2014.pdf?id=9&tra
de=i&id=9&trade=i

5 Which of these forms of biomass are polymers?


1. Cellulose
2. Lignin
3. Hemicellulose

Select the correct answer using the codes below.


A. 1 and 3 only

prelims.insightsonindia.com 3
Online Prelims TEST - 25 (TEXTBOOK)
( InsightsIAS Mock Test Series for UPSC Preliminary Exam 2020 )

B. 2 only
C. 3 only
D. 1, 2 and 3

Your Answer : D
Correct Answer : D

Answer Justification :

Justification: They are parts of Lignocellulose which refers to plant dry matter (biomass).

It is the most abundantly available raw material on the Earth for the production of biofuels, mainly
bio-ethanol.

It is composed of carbohydrate polymers (cellulose, hemicellulose), and an aromatic polymer


(lignin). These carbohydrate polymers contain different sugar monomers (six and five carbon
sugars) and they are tightly bound to lignin.

Lignocellulosic biomass can be broadly classified into virgin biomass, waste biomass and energy
crops. Virgin biomass includes all naturally occurring terrestrial plants such as trees, bushes and
grass.
th
Q Source: Additional Research: Chapter on Fibres: 6 NCERT

6 Consider the following statements.


Bamboo rice
1. is a special rice that is produced from a dying Bamboo shoot
2. is cultivated only in North-eastern India

Select the correct answer using the codes below.


A. 1 only
B. 2 only
C. Both 1 and 2
D. None of the above

Your Answer :
Correct Answer : A

Answer Justification :

Justification: Statement 1: Bamboo Rice is the ripened seed or ripened fruit of the Bamboo tree or
else Bamboo rice is the fruit of Bamboo trees. The Bamboo rice is produced after flowering stage
followed by fruiting. It is special rice that is produced from a dying Bamboo shoot. Many of us are
unaware of this Bamboo Rice, What is Bamboo Rice made out of, Bamboo Rice Cultivation, Health
benefits of bamboo, How to identify the Bamboo Rice shoot and other queries related to Bamboo
Rice and Bamboo Rice Cultivation. Therefore, in this mostly we have come up with all the
information about bamboo Rice and Bamboo Rice farming.

prelims.insightsonindia.com 4
Online Prelims TEST - 25 (TEXTBOOK)
( InsightsIAS Mock Test Series for UPSC Preliminary Exam 2020 )

Bamboo is a tree that may grow even up to 60 to 100 years. It is one of the perennial trees.
However, the Bamboo tree flowers only at the stage before it dies. But, the Bamboo tree undergoes
mass flowering or gregarious flowering at the age before dying. The edible nature of the bamboo
rice or bamboo fruits is identified from some tribal areas where tribal people used to eat bamboo
rice. Bamboo Rice is a seed that is formed from the flower of the Bamboo tree before the end of its
lifespan.

Statement 2: It came to known from the tribal areas of Kerala nearest to Kanyakumari Forest
ranges of Wayanad Sanctuary. The Wayanad sanctuary where some tribal people lives is a rich
habitat for Bamboo trees in India. Bamboo Rice harvesting and selling is a major source of income
for some groups of the tribe.

Four decades later, bamboo rice shows up in Odisha. The rare variety, which was last harvested in
Chandaka-Dampara Wildlife Sanctuary in 1979, grows twice or thrice in a century.

Q Source:
https://www.insightsonindia.com/wp-content/uploads/2019/05/InsightsonIndia-Apr-2019-Current-Aff
airs.pdf

7 Consider the following statements.


Aadhaar and Other Laws (Amendment) Ordinance, 2019
1. Provides for voluntary use of Aadhaar number in physical or electronic form by authentication or
offline verification with the consent of Aadhaar number holder
2. Permits the concerned entities to perform authentication of Aadhaar only when they are compliant
with the standards of privacy and security specified by the concerned Authority

Select the correct answer using the codes below.


A. 1 only
B. 2 only
C. Both 1 and 2
D. None of the above

Your Answer :
Correct Answer : C

Answer Justification :

Justification: The salient features of the amendments are as follows:

Provides for use of twelve-digit Aadhaar number and its alternative virtual identity to conceal
the actual Aadhaar number of an individual. Gives an option to children who are Aadhaar
number holders to cancel their Aadhaar number on attaining the age of eighteen years.

Provides for voluntary use of Aadhaar number in physical or electronic form by authentication
or offline verification with the consent of Aadhaar number holder.

prelims.insightsonindia.com 5
Online Prelims TEST - 25 (TEXTBOOK)
( InsightsIAS Mock Test Series for UPSC Preliminary Exam 2020 )

Permits the entities to perform authentication only when they are compliant with the
standards of privacy and security specified by the Authority; and the authentication is
permitted under any law made by Parliament or is prescribed to be in the interest of State by
the Central Government.

Allows the use of Aadhaar number for authentication on voluntary basis as acceptable KYC
document under the Telegraph Act, 1885 and the Prevention of Money-laundering Act, 2002.

Prevents denial of services for refusing to, or being unable to, undergo authentication.

Provides for establishment of Unique Identification Authority of India Fund.

Provides for civil penalties, its adjudication, appeal thereof in regard to violations of Aadhaar
Act and provisions by entities in the Aadhaar ecosystem.

Q Source: Current affairs of last year

8 In human body, proteins are assembled from amino acids using information encoded in the

A. ATP
B. Mitochondria
C. Glycogen
D. Genes

Your Answer : D
Correct Answer : D

Answer Justification :

Justification: Proteins are large bio-molecules consisting of one or more long chains of amino acid
residues.

Proteins are assembled from amino acids using information encoded in genes. Each protein has its
own unique amino acid sequence that is specified by the nucleotide sequence of the gene encoding
this protein.

Learning: Proteins perform a vast array of functions within organisms, including catalysing
metabolic reactions, DNA replication, responding to stimuli, and transporting molecules from one
location to another.

Proteins differ from one another primarily in their sequence of amino acids, which is dictated
by the nucleotide sequence of their genes, and which usually results in protein folding into a
specific three-dimensional structure that determines its activity.

prelims.insightsonindia.com 6
Online Prelims TEST - 25 (TEXTBOOK)
( InsightsIAS Mock Test Series for UPSC Preliminary Exam 2020 )

The process of synthesizing a protein from an mRNA template is known as translation.

Q Source: Based on past year UPSC papers

9 The South East Asian nations that lie entirely in the Northern Hemisphere are
1. Philippines
2. Thailand
3. Malaysia
4. Indonesia

Select the correct answer using the codes below.


A. 1 and 2 only
B. 3 and 4 only
C. 1 and 3 only
D. 2 and 4 only

Your Answer : A
Correct Answer : A

Answer Justification :

Justification:

Q Source: Map-based: South-East Asia

prelims.insightsonindia.com 7
Online Prelims TEST - 25 (TEXTBOOK)
( InsightsIAS Mock Test Series for UPSC Preliminary Exam 2020 )

10 India had signed a $250 million loan agreement with the World Bank for the National Rural Economic
Transformation Project (NRETP). Consider the following about it.
1. It aims to help women in rural households shift to a new generation of economic initiatives by
developing viable enterprise for farm and non-farm products.
2. The project will involve developing financial products using digital financial services to help small
producer collectives scale-up and engage with the market.

Select the correct answer using the codes below.


A. 1 only
B. 2 only
C. Both 1 and 2
D. None of the above

Your Answer : C
Correct Answer : C

Answer Justification :

Justification: India has signed a $250 million loan agreement with the World Bank for the National
Rural Economic Transformation Project (NRETP) that aims to help women in rural households shift
to a new generation of economic initiatives by developing viable enterprise for farm and non-farm
products. National Rural Economic Transformation project: The National Rural Economic
Transformation project is additional financing to the $500 million National Rural Livelihoods Project
(NRLP) approved by the World Bank in July 2011.

The project will support enterprise development programs for rural poor women and youth by
creating a platform to access finance including start-up financing options to build their individual or
collectively owned and managed enterprises. The project will involve developing financial products
using digital financial services to help small producer collectives scale-up and engage with the
market. It will also support youth skills development, in coordination with the Deen Dayal
Upadhyaya Grameen Kaushalya Yojana.

Q Source: March Current Affairs: 2019

11 Consider the following statements.


“Nehru Committee Report”
1. was submitted before the rejection of the recommendations of the Simon Commission in an all-party
conference that was held at Mumbai.
2. recommended that India should be given Dominion status and independence within the British
Commonwealth

Which of the above is/are correct?


A. 1 only
B. 2 only
C. Both 1 and 2
D. None of the above

prelims.insightsonindia.com 8
Online Prelims TEST - 25 (TEXTBOOK)
( InsightsIAS Mock Test Series for UPSC Preliminary Exam 2020 )

Your Answer : B
Correct Answer : B

Answer Justification :

Learning: The draft constitution was prepared which was called “Nehru Committee Report“. This
report was submitted on August 28, 1928 at the Lucknow conference of all the parties.

The main points of the Nehru report were as follows:

India would be given Dominion status. This means independence within the British
Commonwealth. India will be a federation which shall have a bicameral legislature at the
centre and Ministry would be responsible to the legislature.

Governor General of India would be the constitutional head of India and will have the same
powers as that of British Crown. There will be no separate electorate. The draft report also
defined the citizenship and fundamental rights.

Q Source: Revision: Freedom movement

12 Consider the following statements.


1. Political parties in India are entitled by the Election Commission to nominate “Star Campaigners”
during General Elections.
2. There is no limit to how many “Star Campaigners” can be assigned by a recognized National or
State party.

Select the correct answer using the codes below.


A. 1 only
B. 2 only
C. Both 1 and 2
D. None of the above

Your Answer : A
Correct Answer : A

Answer Justification :

Justification: Political parties are entitled to nominate “Star Campaigners” during General
Elections. A recognized National or State party can have a maximum of 40 “Star campaigners” and
a registered un-recognised party can nominate a maximum of 20 ‘Star Campaigners”. The travel
expenses of star campaigners are not to be accounted for in the election expense accounts of
candidates of their party.

Election Commission ordered the removal of Anurag Thakur and Parvesh Sahib Singh from the list
of star campaigners of Bhartiya Janata Party for the general elections to the Legislative Assembly of
the NCT of Delhi sometime before.

prelims.insightsonindia.com 9
Online Prelims TEST - 25 (TEXTBOOK)
( InsightsIAS Mock Test Series for UPSC Preliminary Exam 2020 )

Q Source:
https://www.indiatoday.in/elections/delhi-assembly-polls-2020/story/election-commission-bjp-remove
-anurag-thakur-parvesh-sahib-list-star-campaigners-delhi-polls-1641190-2020-01-29

13 Consider the following about some of the policy measures to double the income of farmers and farm
labourers in India.
1. Pradhan Mantri Kisan Samman Nidhi (PM-KISAN) scheme aims to provide a payment of Rs. 6000/-
per year to eligible farmers families.
2. A restructured National Bamboo Mission has been launched recently to promote bamboo plantation
on non forest government as well as private land.
3. Pradhan Mantri Kisan Maan-Dhan Yojana (PM-KMY) provides a minimum fixed pension of Rs. 3000/-
to eligible small and marginal farmers.
4. Bee keeping is being promoted under Mission for Integrated Development of Horticulture (MIDH)
to increase the productivity of crops through pollination and increase honey production.

Select the correct answer using the codes below.


A. 1, 2 and 3 only
B. 1, 2, 3 and 4
C. 1 and 4 only
D. 2 and 3 only

Your Answer : B
Correct Answer : B

Answer Justification :

Background: The Government constituted an Inter-ministerial Committee in April, 2016 to


examine issues relating to “Doubling of Farmers Income” and recommend strategies to achieve the
same; report submitted in 2018. The Committee identified seven sources of income growth viz.,
improvement in crop productivity; improvement in livestock productivity; resource use efficiency or
savings in the cost of production; increase in the cropping intensity; diversification towards high
value crops; improvement in real prices received by farmers; and shift from farm to non-farm
occupations.

Justification: Statement 1: With a view to provide income support to all farmers’ families across
the country, to enable them to take care of expenses related to agriculture and allied activities as
well as domestic needs, the Central Government started a new Central Sector Scheme, namely, the
Pradhan Mantri Kisan Samman Nidhi (PM-KISAN). The scheme aims to provide a payment of Rs.
6000/- per year, in three 4-monthly installments of Rs. 2000/- to the farmers families, subject to
certain exclusions relating to higher income groups.

Statement 3: Further with a view to provide social security net for Small and Marginal Farmers
(SMF) as they have minimal or no savings to provide for old age and to support them in the event of
consequent loss of livelihood, the Government has decided to implement another new Central
Sector Scheme i.e. Pradhan Mantri Kisan Maan-Dhan Yojana (PM-KMY) for providing old age
pension to these farmers. Under this Scheme, a minimum fixed pension of Rs. 3000/- will be
provided to the eligible small and marginal farmers, subject to certain exclusion clauses, on
attaining the age of 60 years.

prelims.insightsonindia.com 10
Online Prelims TEST - 25 (TEXTBOOK)
( InsightsIAS Mock Test Series for UPSC Preliminary Exam 2020 )

Statement 2: Under “Har Medh Par Ped”, agro forestry is being promoted for additional income.
With the amendment of Indian Forest Act, 1927, Bamboo has been removed from the definition of
trees. A restructured National Bamboo Mission has been launched in the year 2018 to promote
bamboo plantation on non forest government as well as private land and emphasis on value
addition, product development and markets.

Statement 4: Bee keeping has been promoted under Mission for Integrated Development of
Horticulture (MIDH) to increase the productivity of crops through pollination and increase the
honey production as an additional source of income of farmers.

Also, giving a major boost to the pro-farmer initiatives, the Government has approved a new
Umbrella Scheme ‘Pradhan Mantri Annadata Aay Sanrakshan Abhiyan (PM-AASHA)’. The Scheme is
aimed at ensuring remunerative prices to the farmers for their produce as announced in the Union
Budget for 2018. This is an unprecedented step taken by Govt. of India to protect the farmers’
income which is expected to go a long way towards the welfare of farmers.

Q Source: http://agriculture.gov.in/

https://pib.gov.in/newsite/PrintRelease.aspx?relid=199099

14 Consider the following important parts of the constitution and the subject that they deal with:
1. Part IVA : Directive Principles
2. Part V : The States
3. Part XI : Relations between the Union and the States

Select the correct answer using the codes below.


A. 1 and 2 only
B. 2 and 3 only
C. 3 only
D. 1 only

Your Answer : D
Correct Answer : C

Answer Justification :

Justification: These are some of the important parts:

Part I The Union and its territory Art. 1 to 4

Part II Citizenship Art. 5 to 11

Part III Fundamental Rights Art. 12 to 35

Part IV Directive Principles Art. 36 to 51

Part IVA Fundamental Duties Art. 51A

Part V The Union Art. 52 to 151

prelims.insightsonindia.com 11
Online Prelims TEST - 25 (TEXTBOOK)
( InsightsIAS Mock Test Series for UPSC Preliminary Exam 2020 )

Part VI The States Art. 152 to 237

Part IX The Panchayats Art. 243 to 243O

Part IXA The Muncipalities Art. 243P to 243ZG

Part X The Scheduled and Tribal Areas Art. 244 to 244A

Part XI Relations between the Union and the States Art. 245 to 263

Part XVI Special provisions relation to certain classes Art. 330 to 342

Q Source: Basics: Indian Polity: M Laxmikanth

15 What is/are the differences between a primary pollutant and a secondary pollutant?
1. A primary pollutant is an air pollutant emitted directly from a source unlike a secondary pollutant.
2. A primary pollutant has a much shorter life in the atmosphere than a secondary pollutant.
3. A primary pollutant is characterized as less potent or harmful than a secondary pollutant.

Select the correct answer using the codes below.


A. 1 only
B. 1 and 2 only
C. 2 and 3 only
D. 1 and 3 only

Your Answer :
Correct Answer : A

Answer Justification :

Justification: A primary pollutant is an air pollutant emitted directly from a source. A secondary
pollutant is not directly emitted as such, but forms when other pollutants (primary pollutants) react
in the atmosphere.

Examples of a secondary pollutant include ozone, which is formed when hydrocarbons (HC) and
nitrogen oxides (NOx) combine in the presence of sunlight; nitrogen dioxide (NO2), which is formed
as nitric oxide (NO) combines with oxygen in the air; and acid rain, which is formed when sulfur
dioxide or nitrogen oxides react with water.

Q Source:
http://ec.europa.eu/health/scientific_committees/opinions_layman/en/indoor-air-pollution/glossary/p
qrs/primary-pollutant-secondary-pollutant.htm

16 With reference to the World Food Programme (WFP), consider the following about it.
1. It is an autonomous inter-governmental organization that provides food related assistance to the
United Nations.
2. It aims to achieve “Zero Hunger in 2030”.

Select the correct answer using the codes below.

prelims.insightsonindia.com 12
Online Prelims TEST - 25 (TEXTBOOK)
( InsightsIAS Mock Test Series for UPSC Preliminary Exam 2020 )

A. 1 only
B. 2 only
C. Both 1 and 2
D. None of the above

Your Answer :
Correct Answer : B

Answer Justification :

Justification: Statement 1: The World Food Programme (WFP) is the food assistance branch of the
United Nations and the world’s largest humanitarian organization addressing hunger and
promoting food security. The WFP strives to eradicate hunger and malnutrition, with the ultimate
goal in mind of eliminating the need for food aid itself. It is a member of the United Nations
Development Group and part of its Executive Committee.

Statement 2: The objectives of the World Food Programme are: 1. Save lives and protect livelihoods
in emergencies. 2. Support food security and nutrition and (re)build livelihoods in fragile settings
and following emergencies. 3. Reduce risk and enable people, communities and countries to meet
their own food and nutrition needs. 4. Reduce under-nutrition and break the inter-generational
cycle of hunger. 5. Zero Hunger in 2030

Q Source: Frequently in news

17 Why did the Indian National Congress (INC) reject the Government of India Act of 1935?
1. The act contained no provision of convening a Constituent Assembly for India.
2. There was no mention of universal adult franchise in the act.

Which of the above is/are correct?


A. 1 only
B. 2 only
C. Both 1 and 2
D. None

Your Answer : C
Correct Answer : C

Answer Justification :

Justification: The Act provided for the establishment of an All-India Federation to be based on the
union of the British Indian provinces and Princely States.

The representatives of the States to the federal legislature were to be appointed directly by the
Princes who were to be used to check and counter the nationalists.

The franchise was limited to about one-sixth of the adults.

The Governors, appointed by the British Government, retained special powers. They could veto

prelims.insightsonindia.com 13
Online Prelims TEST - 25 (TEXTBOOK)
( InsightsIAS Mock Test Series for UPSC Preliminary Exam 2020 )

legislative and administrative measures

Q Source: Chapter 25: India’s Struggle for Independence: Bipin Chandra

18 The group that is most susceptible to Malaria among the following is

A. Adolescent females
B. Young children between the age of 5 and 14
C. Adults above the age of 55
D. Children under the age of 5 and pregnant women

Your Answer : D
Correct Answer : D

Answer Justification :

Justification: Some population groups are at considerably higher risk of contracting malaria, and
developing severe disease, than others.

These include pregnant women, infants, children under 5 years of age and patients with HIV/AIDS,
as well as non-immune migrants, mobile populations and travellers.

Children under the age of 5 years are among the most vulnerable to malaria infection as they have
not yet developed any immunity to the disease

A pregnant woman’s risk of infection increases due to changes in her hormone levels and immune
system. First-time mothers are especially vulnerable. Pregnant women suffering from malaria are at
increased risk of anemia and miscarriage, and their babies are at risk of stillbirth, prematurity,
intrauterine growth retardation, and low birth weight.

National malaria control programmes need to take special measures to protect these population
groups from malaria infection, taking into consideration their specific circumstances.

Q Source: https://www.who.int/malaria/areas/high_risk_groups/en/

19 Consider the following about the Bangladesh-Bhutan-India-Nepal (BBIN) initiative.


1. The main objective of BBIN is to facilitate seamless cross-border movement of vehicles.
2. Asian Development Bank (ADB) is providing technical, advisory, and financial support to the BBIN.
3. The initiative also aims to phase out all custom duties and tariff on international trade in goods and
services between the member countries.

Select the correct answer using the codes below.


A. 1 only
B. 2 and 3 only
C. 1, 2 and 3
D. 1 and 2 only

prelims.insightsonindia.com 14
Online Prelims TEST - 25 (TEXTBOOK)
( InsightsIAS Mock Test Series for UPSC Preliminary Exam 2020 )

Your Answer : D
Correct Answer : D

Answer Justification :

Justification: As per the agreement, member countries would allow vehicles registered in the other
countries to enter their territory under certain terms and conditions. Customs and tariffs will be
decided by the respective countries and these would be finalised at bilateral and trilateral forums.

Objective: The main objective of the agreement is to provide seamless people-to-people contact and
enhance economic interaction by facilitating cross border movement of people and goods.

Benefits: It would permit unhindered movement of passenger and cargo vehicles among the four
countries. Cargo vehicles do not have to be changed at the border, a practice that has prevailed
until now. The BBIN agreement will promote safe, economical efficient and environmentally sound
road transport in the subregion and will further help each country in creating an institutional
mechanism for regional integration. www.insightsonindia.com 68 InsightsIAS

Assistance from ADB: The Asian Development Bank (ADB) has been providing technical, advisory,
and financial support to the BBIN MVA initiative as part of its assistance to the South Asia Sub
regional Economic Cooperation (SASEC) program, a projects-based economic cooperation initiative
that brings together the BBIN countries, Maldives, Sri Lanka and more recently, Myanmar. ADB is
the secretariat of SASEC.

Q Source:
https://www.insightsonindia.com/wp-content/uploads/2019/05/InsightsonIndia-Apr-2019-Current-Aff
airs.pdf

20 Consider the following statements.


1. No questions to private Members of Parliament can be raised in the question hour in the
Parliament.
2. No private member can introduce a bill in the house without the prior consent of the Leader of the
House.

Which of the above is/are correct?


A. 1 only
B. 2 only
C. Both 1 and 2
D. None

Your Answer : D
Correct Answer : D

Answer Justification :

Justification: Statement 1: A Question may also be addressed to a Private Member under Rule 40
of the Rules of Procedure and Conduct of Business in Lok Sabha, which provides that the subject
matter of the question should be related to some Bill, Resolution or other matter connected with the

prelims.insightsonindia.com 15
Online Prelims TEST - 25 (TEXTBOOK)
( InsightsIAS Mock Test Series for UPSC Preliminary Exam 2020 )

Business of the House for which that Member is responsible.

The procedure in regard to such questions is the same as that followed in the case of questions
addressed to a Minister with such variations as the Speaker may consider necessary.

Statement 2: No such consent in required.

Q Source: http://164.100.47.194/loksabha/FAQ.aspx

21 “Asian Development Outlook” report is published by

A. Asian Development Bank


B. World Bank
C. International Monetary Fund
D. Partnership for Asia Network

Your Answer : D
Correct Answer : A

Answer Justification :

Justification: ADB published the Asian Development Outlook 2019.

Key findings: Growth in developing Asia is projected to soften to 5.7% in 2019 and 5.6% in 2020.
Excluding Asia’s high income newly industrialized economies, growth is expected to slip from 6.4%
in 2018 to 6.2% in 2019 and 6.1% in 2020.

Since oil prices rose and Asian currencies depreciated, inflation edged up last year but remained
low by historical standards. In light of stable commodity prices, inflation is anticipated to remain
subdued at 2.5% in both 2019 and 2020

Q Source:
https://www.insightsonindia.com/wp-content/uploads/2019/05/InsightsonIndia-Apr-2019-Current-Aff
airs.pdf

22 Consider the following statements.


1. FAME India is a part of the National Electric Mobility Mission Plan.
2. Main thrust of FAME is to encourage electric vehicles by providing subsidies.

Select the correct answer using the codes below.


A. 1 only
B. 2 only
C. Both 1 and 2
D. None of the above

Your Answer : D
Correct Answer : C

prelims.insightsonindia.com 16
Online Prelims TEST - 25 (TEXTBOOK)
( InsightsIAS Mock Test Series for UPSC Preliminary Exam 2020 )

Answer Justification :

Justification: Salient features of FAME 2 scheme:

1. Faster Adoption and Manufacturing of Hybrid and Electric Vehicles, or FAME 2 scheme aims to
boost electric mobility and increase the number of electric vehicles in commercial fleets.

2. Target: The outlay of ₹10,000 crore has been made for three years till 2022 for FAME 2 scheme.

3. The government will offer the incentives for electric buses, three-wheelers and four-wheelers to
be used for commercial purposes. 4. Plug-in hybrid vehicles and those with a sizeable lithium-ion
battery and electric motor will also be included in the scheme and fiscal support offered depending
on the size of the battery

Q Source:
https://www.insightsonindia.com/wp-content/uploads/2019/05/InsightsonIndia-Apr-2019-Current-Aff
airs.pdf

23 Under the Government of India (Allocation of Business) Rules 1961, which of the following is the
nodal ministry for all matters of Union Territories relating to legislation, finance and budget,
services?

A. Prime Minister’s Office (PMO)


B. Ministry of Urban Development
C. Ministry of Finance
D. Ministry of Home Affairs

Your Answer : D
Correct Answer : D

Answer Justification :

Learning: Apart from the nodal ministry provision, all the five UTs without a legislature (Andaman
and Nicobar Islands, Chandigarh, Daman and Diu, Dadra and Nagar Haveli, and Lakshadweep)
have the forum of Home Minister’s Advisory Committee (HMAC).

On this committee, besides the Administrator and Mp from the respective UT, members from the
local elected bodies, e.g., District Panchayats and Municipal Council / Committees are nominated as
members.

Meetings of the HMAC are chaired by the Union Home Minister, or, in his absence, by the Minister
of State in the Ministry of Home Affairs.

The Committee discusses the general issues relating to social and economic development of the
Union territories.

Q Source: Chapter 36: Indian Polity: M Laxmikanth

prelims.insightsonindia.com 17
Online Prelims TEST - 25 (TEXTBOOK)
( InsightsIAS Mock Test Series for UPSC Preliminary Exam 2020 )

24 Consider the following about EMISAT satellite, launched by ISRO.


1. It can intercept signals broadcasted by communication systems, radars, and some other electronic
systems.
2. It is primarily meant for non-electromagnetic spectrum (non-EM) measurements, but can also
measure EM spectrum.

Select the correct answer using the codes below.


A. 1 only
B. 2 only
C. Both 1 and 2
D. None of the above

Your Answer :
Correct Answer : A

Answer Justification :

Justification: EMISAT is an advanced electronic intelligence (ELINT) satellite jointly developed by


ISRO-DRDO. It is meant for electromagnetic spectrum measurements. It is modelled after a famous
Israeli spy satellite called SARAL (Satellite with ARgos and ALtika). Both these satellites have the
SSB-2 bus protocol — the core component for their sharp electronic surveillance capabilities across
the length and width of a large country like India.

EMISAT also has a special altimeter (a radar altitude measuring device) called ‘AltiKa’ that works in
the Ka band microwave region of the spectrum. The electronic surveillance payload of EMISAT was
developed under a DRDO’s project called KAUTILYA.

The main capability of EMISAT is in signal intelligence — intercepting signals broadcasted by


communication systems, radars, and other electronic systems. The Ka-band frequency that EMISAT
is sensitive to, allows the 436-kg EMISAT — India’s newest spy in the sky — to scan through ice,
rain, coastal zones, land masses, forests and wave heights with ease.

Q Source:
https://www.insightsonindia.com/wp-content/uploads/2019/05/InsightsonIndia-Apr-2019-Current-Aff
airs.pdf

25 Pradhan Mantri Annadata Aay Sanrakshan Abhiyan (PM-AASHA) ensures Minimum Support Price
(MSP) to farmers for

A. Forest produce
B. Oilseeds and pulses
C. Fruits and vegetables
D. Cereals

Your Answer : D
Correct Answer : B

prelims.insightsonindia.com 18
Online Prelims TEST - 25 (TEXTBOOK)
( InsightsIAS Mock Test Series for UPSC Preliminary Exam 2020 )

Answer Justification :

Justification: Pradhan Mantri Annadata Aay Sanrakshan Abhiyan (PM-AASHA) ensures Minimum
Support Price (MSP) to farmers of notified oilseeds and pulses qualifying Fair Average Quality
(FAQ) norms.

PM-AASHA is an umbrella scheme comprising of Price Support Scheme (PSS), Price Deficiency
Payment Scheme (PDPS) and Private Procurement & Stockist Scheme (PPSS). These schemes are
implemented at the request of the State Governments / Union Territories.

PSS is implemented for procurement of pulses, oilseeds and copra at MSP, whereas PDPS is
implemented for oilseeds. However, States/UTs may choose either PSS or PDPS in a given
procurement season with respect to a particular oilseed crop for the entire State.

Q Source: http://agriculture.gov.in/

26 Which of the following has formed the largest chunk of the Union Government expenditure in the
annual budget of last five years?

A. Salaries (pay & allowances)


B. Interest payments
C. Total subsidies
D. Pensions

Your Answer : B
Correct Answer : B

Answer Justification :

Justification:

prelims.insightsonindia.com 19
Online Prelims TEST - 25 (TEXTBOOK)
( InsightsIAS Mock Test Series for UPSC Preliminary Exam 2020 )

Q Source: Vol 2: Economic Survey 2018-19

27 A Capitalist society usually has


1. A deregulated price system
2. Largely free entry and exit of firms in a competitive market
3. No public property

Select the correct answer using the codes below.


A. 1 only
B. 1 and 2 only
C. 2 and 3 only
D. 1 and 3 only

Your Answer : B
Correct Answer : B

Answer Justification :

Justification: Statement 1 and 2: Private property, capital accumulation, wage labour, voluntary
exchange, a price system and competitive markets are some of the major features of capitalism.

In a capitalist market economy, decision-making and investment are determined by every owner of
wealth, property or production ability in financial and capital markets, whereas prices and the
distribution of goods and services are mainly determined by competition in goods and services
markets.

Statement 3: The government can still own property in capitalistatic economies, its just that most of
the property is private property. Public property still exists.

But, largely Capitalism is an economic system based upon private ownership of the means of
production and their operation for profit.

Q Source: General questions on Economic systems

28 Consider the following statements about Kanyashree scheme.


1. It is a conditional cash transfer scheme aiming at improving the status and wellbeing of the girl

prelims.insightsonindia.com 20
Online Prelims TEST - 25 (TEXTBOOK)
( InsightsIAS Mock Test Series for UPSC Preliminary Exam 2020 )

child by incentivising schooling of teenage girls and delaying their marriages until the age of 18.
2. The scheme received the United Nations Public Service Award.

Select the correct answer using the codes below.


A. 1 only
B. 2 only
C. Both 1 and 2
D. None of the above

Your Answer : C
Correct Answer : C

Answer Justification :

Justification: Kanyashree is a conditional cash transfer scheme aiming at improving the status and
wellbeing of the girl child by incentivising schooling of teenage girls and delaying their marriages
until the age of 18. It received the United Nations Public Service Award in 2018.

Performance of the scheme: Through the initiative, cash was deposited into the bank account of
girls for every year they remained in school and were unmarried. This initiative led to a “drastic
reduction in child marriage, increase in female education and female empowerment.”

Experts and activists have observed that Kanyashree stipends are no shield against trafficking. They
say the complex problem cannot be prevented merely by monetary handouts meant to retain girls in
school.

Q Source:
https://www.insightsonindia.com/wp-content/uploads/2019/04/InsightsonIndia-Mar-2019-Current-Aff
airs.pdf

29 Which of the following are examples of Chemical Changes?


1. Cooking an egg
2. Digesting food
3. Heating sugar to form caramel
4. Baking a cake

Select the correct answer using the codes below.


A. 1 and 3 only
B. 2, 3 and 4 only
C. 1, 2 and 4 only
D. 1, 2, 3 and 4

Your Answer : B
Correct Answer : D

Answer Justification :

Justification: No new substance is created during a physical change, although the matter takes a

prelims.insightsonindia.com 21
Online Prelims TEST - 25 (TEXTBOOK)
( InsightsIAS Mock Test Series for UPSC Preliminary Exam 2020 )

different form. In a chemical change a new substance is created and old one is modified.

One way to identify a physical change is that such a changes may be reversible, especially phase
changes. A chemical change is difficult to reverse.

For example, if you freeze an ice cube, you can melt it into the water again.

Examples of Physical Changes are

crushing a can

melting an ice cube

boiling water

mixing sand and water

Examples of Chemical changes are

Souring milk.

Mixing acid and base.

Digesting food.

Cooking an egg.

Heating sugar to form caramel.

Baking a cake.

Rusting of iron.

th
Q Source Additional Research: Chapter 6: 6 NCERT: Science

30 National sample Survey (NSS), that gives useful information on consumer expenditure, employment
and the like, is conducted by the

A. NITI Aayog, and formerly Planning Commission

prelims.insightsonindia.com 22
Online Prelims TEST - 25 (TEXTBOOK)
( InsightsIAS Mock Test Series for UPSC Preliminary Exam 2020 )

B. Ministry of Statistics and Programme Implementation


C. Department of Economic Affairs, Ministry of Finance
D. Department of Industrial Policy and Promotion, Ministry of Commerce

Your Answer : B
Correct Answer : B

Answer Justification :

Learning: The Employment and Unemployment surveys of National sample Survey (NSS) are the
primary sources of data on various indicators of labour force at National and State levels.

These are used for planning, policy formulation, decision support and as input for further statistical
exercises by various Government organizations, academicians, researchers and scholars.

NSS surveys on employment and un-employment with large sample size of households have been
conducted quinquennially from 1972 of NSS onwards.

Q Source: Page 131: 11th NCERT: Indian Economic Development

31 Consider the following statements.


The Commonwealth of Nations
1. is an intergovernmental organisation
2. consists of Russian Federation and its former satellite states
3. is an agency dedicated to the security of its members

Select the correct answer using the codes below.


A. 1 and 2 only
B. 1 only
C. 2 and 3 only
D. 1 and 3 only

Your Answer : B
Correct Answer : B

Answer Justification :

Justification: Formerly the British Commonwealth, it is an intergovernmental organisation of 53


member states that are mostly former territories of the British Empire.

It dates back to the mid-20th century with the decolonisation of the British Empire through
increased self-governance of its territories.

It was formally constituted by the London Declaration in 1949, which established the member states
as "free and equal"

The Commonwealth operates by intergovernmental consensus of the member states.

Q Source: International bodies and organizations


prelims.insightsonindia.com 23
Online Prelims TEST - 25 (TEXTBOOK)
( InsightsIAS Mock Test Series for UPSC Preliminary Exam 2020 )

32 Khajuraho Temples were built in

A. Ancient India by Gupta rulers


B. Medieval India by Chandella rulers
C. The Iron age by Pala rulers
D. The Age of Wanderers by Eastern Ganga dynasty

Your Answer : B
Correct Answer : B

Answer Justification :

Learning: These temples were built by the Chandella ruler between AD 900 and 1130.

They are world-wide known for their erotic sculptures. The first recorded mention of the Khajuraho
temples is in the accounts of Abu Rihan al Biruni in AD 1022 and the Arab traveler Ibn Battuta in
AD 1335.

The Khajuraho group of temples were built together but were dedicated to two religions, Hinduism
and Jainism, suggesting a tradition of acceptance and respect for diverse religious views among
Hindus and Jains in the region.

Q Source: Based on past year UPSC papers

33 Rules of origin (ROO) are the criteria needed to determine the origin of a product for purposes of
international trade. They are also used for determining
1. Whether the imported product shall receive preferential treatment
2. Whether the rules of government procurement may apply to the imported product

Select the correct answer using the codes below.


A. 1 only
B. 2 only
C. Both 1 and 2
D. None of the above

Your Answer : A
Correct Answer : C

Answer Justification :

Justification: Their importance is derived from the fact that duties and restrictions in several cases
depend upon the source of imports.

Rules of origin are used: to implement measures and instruments of commercial policy such as
antidumping duties and safeguard measures; to determine whether imported products shall receive
most-favoured-nation (MFN) treatment or preferential treatment; for the purpose of trade statistics;
for the application of labelling and marking requirements; and for government procurement.

prelims.insightsonindia.com 24
Online Prelims TEST - 25 (TEXTBOOK)
( InsightsIAS Mock Test Series for UPSC Preliminary Exam 2020 )

Q Source:
https://commerce.gov.in/writereaddata/trade/FAQ_on_FTA_9April2014.pdf?id=9&am
p;trade=i&id=9&trade=i

34 With reference to Scheduled areas, consider the following statements.


1. The executive power of a state does not extend to the scheduled areas therein.
2. The executive power of the Centre extends to giving directions to the states regarding the
administration of such areas.
3. The Governor of the State is obliged to submit an annual report to the president regarding the
administration of such areas.

Select the correct answer using the codes below.


A. 1 and 2 only
B. 2 and 3 only
C. 1 and 3 only
D. 2 only

Your Answer : B
Correct Answer : B

Answer Justification :

Justification: The executive power of the stat extends to these areas.

The governor is empowered to direct that any particular act of Parliament or the state
legislature does not apply to a scheduled area or apply with specified modifications and
exceptions.

He can also make regulations for the peace and good government of a scheduled area after
consulting the tribes advisory council.

Such regulations also prohibit or restrict the transfer of land by or among members of the
scheduled tribes, regulate the allotment of land to members of the scheduled tribes and
regulate the business of money-lending in relation to the scheduled tribes.

Also, a regulation may repeal or amend any act of Parliament or the state legislature, which is
applicable to a scheduled area. But, all such regulations require the assent of the president.

Q Source: Chapter 36: Indian Polity: M Laxmikanth

35 Golan Heights, sometimes seen in news, is a part of the territory of

A. Turkey

prelims.insightsonindia.com 25
Online Prelims TEST - 25 (TEXTBOOK)
( InsightsIAS Mock Test Series for UPSC Preliminary Exam 2020 )

B. Egypt
C. Syria
D. Iraq

Your Answer : C
Correct Answer : C

Answer Justification :

Justification: It is a region in Levant, Syria internationally recognized as Syrian territory but


occupied by Israel.

US President Donald Trump has backed Israeli sovereignty over the Golan Heights, seized from
Syria in 1967.

The Golan announcement is likely to further complicate Trump’s long-awaited plan to resolve the
Israeli-Palestinian conflict.

Why is this area contentious?

The Golan Heights were part of Syria until 1967, when Israel captured most of the area in the Six
Day War, occupying it and annexing it in 1981. That unilateral annexation was not recognised
internationally, and Syria demands the return of the territory.

Syria tried to regain the Heights in the 1973 Middle East war, but was thwarted. Israel and Syria
signed an armistice in 1974 and the Golan had been relatively quiet since. In 2000, Israel and Syria
held their highest-level talks over a possible return of the Golan and a peace agreement. But the
negotiations collapsed and subsequent talks also failed.

Q Source:
https://www.insightsonindia.com/wp-content/uploads/2019/04/InsightsonIndia-Mar-2019-Current-Aff
airs.pdf

36 The following persons cannot become chairperson of Lokpal:


1. A person who holds any office of trust / profit
2. Members of Panchayats or Municipality
3. A person who is affiliated to a political party

Select the correct answer using the codes below.


A. 1 only
B. 1, 2 and 3
C. 1 and 3 only
D. 2 only

Your Answer : A
Correct Answer : B

Answer Justification :

prelims.insightsonindia.com 26
Online Prelims TEST - 25 (TEXTBOOK)
( InsightsIAS Mock Test Series for UPSC Preliminary Exam 2020 )

Justification: The following persons cannot become chairperson of Lokpal:

MPs and MLAs Persons convicted of any offense involving moral turpitude Less than 45 years
of age, Members of Panchayats or Municipality,

A person who was removed or dismissed from the public service,

A person who holds any office of trust / profit; if so, he would need to resign from Lokpal.

A person who is affiliated to a political party Carries on some business / profession; if so, he would
need to quit some business.

Term of Office: The term of office for Lokpal Chairman and Members is 5 years or till attaining age
of 70 years. The salary, allowances and other conditions of service of chairperson are equivalent to
Chief Justice of India and members is equivalent to Judge of Supreme Court. If the person is already
getting the pension (for being a former judge), the equivalent pension amount will be deducted from
the salary. The source of salary for Lokpal and Members is Consolidated Fund of India. If the
chairperson dies in office or has resigned from the post, President can authorise the senior-most
Member to act as the Chairperson until new chairperson is appointed. If chairperson is not available
for certain functions due to leave, his job will be done by senior most member.

Q Source: In news frequently

37 Among the following Acts, an Enforcement Directorate (ED) in India will be most concerned with

A. Narcotic Drugs and Psychotropic Substances Act


B. Negotiable Instrument Act
C. Public Debt Act
D. Prevention of Money Laundering Act

Your Answer : D
Correct Answer : D

Answer Justification :

Learning: It is a law enforcement agency and economic intelligence agency responsible for
enforcing economic laws and fighting economic crime in India. It is part of the Department of
Revenue, Ministry of Finance.

The prime objective of the Enforcement Directorate is the enforcement of two key Acts- the Foreign
Exchange Management Act 1999 (FEMA) and the Prevention of Money Laundering Act 2002
(PMLA).

Other objectives are primarily linked to checking money laundering in India.

prelims.insightsonindia.com 27
Online Prelims TEST - 25 (TEXTBOOK)
( InsightsIAS Mock Test Series for UPSC Preliminary Exam 2020 )

It comprises officers of the Indian Revenue Service, Indian Police Service and the Indian
Administrative Service.

Q Source: Frequently in news

38 With reference to Real Estate Investment Trusts (REITs), consider the following statements.
1. REITs allow one to invest in income-generating real estate assets.
2. Securities and Exchange Board of India (SEBI) mandated that all REITS be listed on exchanges and
make an initial public offer to raise money.

Select the correct answer using the codes below.


A. 1 only
B. 2 only
C. Both 1 and 2
D. None of the above

Your Answer :
Correct Answer : C

Answer Justification :

Justification: REITs are similar to mutual funds. While mutual funds provide for an opportunity to
invest in equity stocks, REITs allow one to invest in income-generating real estate assets.

They are collective investment vehicles that operate and manage property portfolios and give
returns to investors. Securities and Exchange Board of India (SEBI) mandated that all REITS be
listed on exchanges and make an initial public offer to raise money.

There are three types of REIT available: equity REITs which purchase, own and manage income-
generating properties; mortgage REITs which lend money directly or indirectly to real estate
owners; and hybrid REITs which are a combination of the first two.

The trusts are listed in stock exchanges so that investors can buy units in the trust. REITs are
structured as trusts. Thus, the assets of an REIT are held by an independent trustee on behalf of
unit holders.

Q Source:
https://www.insightsonindia.com/wp-content/uploads/2019/04/InsightsonIndia-Mar-2019-Current-Aff
airs.pdf

39 What kind of mathematical relationship obtains between the amount of stratospheric ozone and the
intensity of the solar UVR measured at the Earth’s surface?

A. Direct
B. Inverse
C. No relationship
D. Either B or C, depending on the attitude

prelims.insightsonindia.com 28
Online Prelims TEST - 25 (TEXTBOOK)
( InsightsIAS Mock Test Series for UPSC Preliminary Exam 2020 )

Your Answer : B
Correct Answer : B

Answer Justification :

Justification & Learning: The ozone layer acts as a filter for the shorter wavelength and highly
hazardous ultraviolet radiation (UVR) from the sun, protecting life on Earth from its potentially
harmful effects.

Ozone absorbs this UV radiation and breaks down to oxygen. Oxygen is different from ozone.

There is an inverse relationship between stratospheric ozone and solar UVR measured at the
Earth’s surface, i.e. the lower the ozone levels, the higher the solar UVR. The formation of ozone
holes severely affects ozone’s ability to filter UV rays.

Q Source: Page 63: VIIth Science

40 Consider the following statements.


1. ‘Pradhan Mantri Bhartiya Janaushadhi Pariyojana’ aims to provide quality medicines at affordable
prices to the masses from any retail outlet in India.
2. Pradhan Mantri Bhartiya Jan Aushadhi Kendra (PMBJK) have been set up across India to provide
generic drugs to the needy.

Select the correct answer using the codes below.


A. 1 only
B. 2 only
C. Both 1 and 2
D. None of the above

Your Answer : B
Correct Answer : B

Answer Justification :

Justification: Statement 1: ‘Pradhan Mantri Bhartiya Janaushadhi Pariyojana’ is a campaign


launched by the Department of Pharmaceuticals, Govt. Of India, to provide quality medicines at
affordable prices to the masses through special kendra’s known as Pradhan Mantri Bhartiya Jan
Aushadhi Kendra.

Statement 2: Pradhan Mantri Bhartiya Jan Aushadhi Kendra (PMBJK) have been set up to provide
generic drugs, which are available at lesser prices but are equivalent in quality and efficacy as
expensive branded drugs. Bureau of Pharma PSUs of India (BPPI) is the implementing agency of
PMBJP. BPPI (Bureau of Pharma Public Sector Undertakings of India) has been established under
the Department of Pharmaceuticals, Govt. of India, with the support of all the CPSUs.

Q Source: March Current Affairs: 2019

41 In a simple Keynesian model of the economy, Aggregate Demand (AD) usually increases with an increase in

prelims.insightsonindia.com 29
Online Prelims TEST - 25 (TEXTBOOK)
( InsightsIAS Mock Test Series for UPSC Preliminary Exam 2020 )

1. National Income
2. Investment
3. Government spending
4. Rise in population

Select the correct answer using the codes below.


A. 1, 2 and 3 only
B. 2 and 3 only
C. 1 and 3 only
D. 1 and 4 only

Your Answer :
Correct Answer : A

Answer Justification :

Justification: Under the effective demand principle, the equilibrium output of the final goods is
equal to ex ante aggregate demand.

Thus, a basic equation used for determining aggregate demand is

AD = Y = C + I + G, where C is usually an increasing function of national income. Y is total output.

Based on the above, if any of the factors are increased, it increases the AD.
th
Q Source: Chapter 4: 12 Macroeconomics NCERT

42 Who among the following, on ceasing to hold office, is/are eligible for further employment in the
Government of India?

A. Election Commissioners of India


B. Member, UPSC
C. Comptroller and Auditor General of India
D. All of the above

Your Answer : A
Correct Answer : A

Answer Justification :

Justification: Option A: The Constitution has not debarred the retiring election commissioners
from any further appointment by the government.

Option B: It would be appropriate to mention here that the chairman and members of UPSC and
State Public Service Commission are ineligible under Article 319 for further employment under the
government of India or any state government after retirement, except in those very commissions
as chairman.

prelims.insightsonindia.com 30
Online Prelims TEST - 25 (TEXTBOOK)
( InsightsIAS Mock Test Series for UPSC Preliminary Exam 2020 )

Option C: The CAG is not eligible for further office either under the Government of India or under
the Government of any State after he has ceased to hold his office. These provisions are in order to
ensure the independence of CAG.

Q Source: Chapter 38: Indian Polity: M Laxmikanth

43 Consider the following statements about some of the fiscal parameters of the Government of India.
1. Gross tax revenue has steadily increased year-on-year in the last five years.
2. Revenue expenditure has declined steadily year-on-year from 2011 onwards.

Select the correct answer using the codes below.


A. 1 only
B. 2 only
C. Both 1 and 2
D. None of the above

Your Answer : A
Correct Answer : A

Answer Justification :

Q Source: Vol 2: Economic Survey 2018-19

44 Consider the following statements.


1. A Photon sphere is created around black holes because light changes its wavelength as it nears a
black hole.

prelims.insightsonindia.com 31
Online Prelims TEST - 25 (TEXTBOOK)
( InsightsIAS Mock Test Series for UPSC Preliminary Exam 2020 )

2. Event Horizon is the brightest and most visible part of a black hole.
3. Singularity is observed at the centre of a black hole.

Select the correct answer using the codes below.


A. 1, 2 and 3
B. 2 and 3 only
C. 1 and 2 only
D. 3 only

Your Answer :
Correct Answer : D

Answer Justification :

Justification: S3: The "singularity" is at the center of the black hole where all inside materials
head towards and whose density and gravity approaches infinitely. The appearance of a singularity
in General Relativity indicates the limit of this theory. It illustrates places and/or conditions where
the theory breaks down and one can not know the details of exactly what is happening.

S1: Its wrong. Light does not change its wavelength nearing a black hole.

The red ring above labeled the photon sphere is also called the photon ring. The photon sphere is
also called the innermost stable circular orbit (ISCO). This is another mathematical theoretical
boundary. As the gas and other material in the accretion disk spin closer and closer to the black
hole, they travel faster and faster approaching the speed of light. The photon sphere is the point
where only light photons can can travel fast enough at the full speed of light to continuously orbit
the black hole.

S2: The event horizon is the threshold around the black hole where the escape velocity surpasses
the speed of light. According to Einstein's theory of special relativity, nothing can travel faster
through space than the speed of light.

prelims.insightsonindia.com 32
Online Prelims TEST - 25 (TEXTBOOK)
( InsightsIAS Mock Test Series for UPSC Preliminary Exam 2020 )

According to Einstein's theory of special relativity, nothing can travel faster through space than the
speed of light. This means a black hole's event horizon is essentially the point from which nothing
can return. The name refers to the impossibility of witnessing any event taking place inside that
border, the horizon beyond which one cannot see.

Q Source:
https://www.insightsonindia.com/wp-content/uploads/2019/05/InsightsonIndia-Apr-2019-Current-Aff
airs.pdf

45 Policy corridor in Monetary policy refers to the difference between

A. Base rate and Marginal Cost of Lending Rate (MCLR)


B. Repo rate and MCLR
C. Reverse repo rate and Marginal Standing Facility (MSF) rate
D. Repo rate and Cash Reserve Ratio (CRR)

Your Answer : D
Correct Answer : C

Answer Justification :

Justification: Corridor in monetary policy refers to the difference between the reverse repo rate
and the high cost MSF rate. At reverse repo rate, banks can park funds with RBI. As MSF rate, they
can borrow in urgent situations from RBI to meet short-term liquidity needs. So, on one end, we
have a rate that RBI is willing to give to banks (which is usually lower than other rates prevailing in
the market), and on the other end, we have repo rate and MSF (slightly higher than repo rate), at
which banks can borrow from RBI.

Ideally, the call rate should travel within the corridor showing a comfortable liquidity situation in
the financial system and economy. The corridor structure for the policy rate is a helping guide for
the RBI to design its monetary policy operations.

Call money rate is the operating target of monetary policy. As far as it lies within the corridor, there
is not much liquidity disturbance in the system.

Q Source: Vol 2: Economic Survey 2018-19

46 Pradhan Mantri Krishi Sinchayee Yojana (PMKSY-PDMC) is operational in the country from 2015-16.
Consider the following about it.
1. The PMKSY- PDMC focuses on enhancing water use efficiency at farm level through Micro
Irrigation.
2. The Government provides financial assistance exclusively for small and marginal farmers
installation of Drip and Sprinkler Irrigation systems.

Select the correct answer using the codes below.


A. 1 only
B. 2 only

prelims.insightsonindia.com 33
Online Prelims TEST - 25 (TEXTBOOK)
( InsightsIAS Mock Test Series for UPSC Preliminary Exam 2020 )

C. Both 1 and 2
D. None of the above

Your Answer : A
Correct Answer : A

Answer Justification :

Justification: Per Drop More Crop component of Pradhan Mantri Krishi Sinchayee Yojana (PMKSY-
PDMC) is operational in the country from 2015-16. The PMKSY- PDMC focuses on enhancing water
use efficiency at farm level through Micro Irrigation viz. Drip and Sprinkler irrigation.

The Government provides financial assistance @ 55% for small and marginal farmers and @
45% for other farmers for installation of Drip and Sprinkler Irrigation systems. In addition,
some States provide additional incentives/top up subsidy for encouraging farmers to adopt
Micro Irrigation.

Wide publicity through press & print media, publication of leaflets/booklets, organization of
workshops, exhibitions, farmer fairs, information on State/Government of India web portals
etc. is being done. Demonstrations have been setup in 160 KrishiVigyanKendras (KVKs) for
educating the farmers.

42.15 lakh ha (Drip 22.81 lakh ha & Sprinkler 19.34 lakh ha) has been covered under PMKSY-
PDMC since 2015-16 till date.

Q Source: http://agriculture.gov.in/

https://pib.gov.in/newsite/PrintRelease.aspx?relid=199256

47 Bomb cyclone, recently seen in news

A. is caused by massive anthropogenic explosions in the Sea


B. is a mid-latitude cyclone that intensifies rapidly
C. forms in regions of high pressure in the Northern hemisphere
D. None of the above is correct.

Your Answer : B
Correct Answer : B

Answer Justification :

Justification: A powerful bomb cyclone has impacted several part of the US causing heavy floods.

Bomb cyclone is a term used by meteorologists to indicate a mid-latitude cyclone that intensifies
rapidly.

prelims.insightsonindia.com 34
Online Prelims TEST - 25 (TEXTBOOK)
( InsightsIAS Mock Test Series for UPSC Preliminary Exam 2020 )

A bomb cyclone happens when atmospheric pressure in the middle of the storm drops at least 24
millibars over 24 hours, quickly increasing in intensity. The lower the pressure, the stronger the
storm.

How it works?

Deep drops in barometric pressure occur when a region of warm air meets one of cold air. The air
starts to move and the rotation of the earth creates a cyclonic effect. The direction is counter
clockwise in the Northern hemisphere leading to winds that come out of the northeast.

Learning: What’s the difference between hurricanes, cyclones and typhoons?

Hurricanes, cyclones and typhoons are all tropical storms. They are all the same thing but are given
different names depending on where they appear. When they reach populated areas they usually
bring very strong wind and rain which can cause a lot of damage. Hurricanes are tropical storms
that form over the North Atlantic Ocean and Northeast Pacific. Cyclones are formed over the South
Pacific and Indian Ocean. Typhoons are formed over the Northwest Pacific Ocean

Q Source:
https://www.insightsonindia.com/wp-content/uploads/2019/04/InsightsonIndia-Mar-2019-Current-Aff
airs.pdf

48 Non Agricultural Market Access (NAMA) relates to trade negotiations on non-agricultural or


industrial products. The product coverage under NAMA includes
1. Marine products
2. Textile and clothing
3. Horticultural products

Select the correct answer using the codes below.


A. 1 and 2 only
B. 2 and 3 only
C. 1, 2 and 3
D. It excludes it all.

Your Answer : D
Correct Answer : A

Answer Justification :

Justification: In the NAMA negotiations, WTO Members discuss the terms or modalities for
reducing or eliminating customs tariff and non tariff barriers on trade in industrial products.

The product coverage under NAMA includes marine products, chemicals, rubber products, wood
products, textiles and clothing, leather, ceramics, glassware, engineering products, electronics,
automobiles, instruments, sports goods and toys.

On tariffs, the negotiations take place on the bound tariff which are the bindings taken during the
negotiations at the WTO. The bound tariffs are the upper limit of the applied customs tariff which

prelims.insightsonindia.com 35
Online Prelims TEST - 25 (TEXTBOOK)
( InsightsIAS Mock Test Series for UPSC Preliminary Exam 2020 )

are the tariffs actually applied by the Customs authorities on imports into any country.

In the NAMA negotiations there are tariffs on which no bindings have been taken and these are
known as the unbound tariff lines. Based on the commitments taken by India, at the commencement
of the Doha Round in 2001, India has more than 31% of it NAMA tariff lines as unbound.

Elements of NAMA Negotiations:

The main elements of the NAMA negotiations are:

(i) Coefficient for the tariff reduction formula

(ii) Flexibilities for protecting sensitive NAMA products

(iii) Sectoral initiatives for elimination of customs tariff in specific sectors

(iv) Non-Tariff Barrier (NTB) textual proposals

Q Source: https://commerce.gov.in/PageContent.aspx?Id=75

49 With reference to Agriculture Skill Council of India (ASCI), consider the following statements.
1. It is a not-for-profit organization.
2. It works under the aegis of Ministry of Agriculture and Farmer’s Welfare.

Select the correct answer using the codes below.


A. 1 only
B. 2 only
C. Both 1 and 2
D. None of the above

Your Answer :
Correct Answer : A

Answer Justification :

Justification: Agriculture Skill Council of India (ASCI) is a Section 8 Not for profit concern working
under the aegis of Ministry of Skill Development & Entrepreneurship (MSDE). ASCI works towards
capacity building by bridging gaps and upgrading skills of farmers, wage workers, self-employed &
extension workers engaged in organized / unorganized segments of Agriculture & Allied Sectors.

ASCI is contributing to nation building through Skill Development in Agriculture especially at the
times when country's agriculture is experiencing stagnant growth, exodus of quality manpower to
other sectors, changing climate with increased variability in production parameters and
transformations in international agriculture markets that are especially too much subsidized
challenging the competitiveness of Indian Agriculture.

Objectives

prelims.insightsonindia.com 36
Online Prelims TEST - 25 (TEXTBOOK)
( InsightsIAS Mock Test Series for UPSC Preliminary Exam 2020 )

Determining skills/competency standards and qualifications and development of National


Occupational Standards (NOS).

Preparation and maintenance of skill inventory to facilitate individual choices.

Development of sector specific skill development plans.

Standardisation of affiliation and accreditation process.

Affiliation, accreditation, assessment and certification of Vocational Institutes/Programmes.

Plan and execute Training of Trainers (ToT).

Promotion of academics of excellence.

Establishment of a well-structured, sector specific, Labour Market Information System (LMIS)


to assist planning and delivery of training.

Adoption of global best practices

Q Source: http://agriculture.gov.in/

http://asci-india.com/about-us.php

50 Consider the following statements.


1. SWIFT India is a joint venture of top Indian public and private sector banks and SWIFT (Society for
Worldwide Interbank Financial Telecommunication).
2. SWIFT India company was created to deliver high quality domestic financial messaging services to
the Indian financial community.

Select the correct answer using the codes below.


A. 1 only
B. 2 only
C. Both 1 and 2
D. None of the above

Your Answer : A
Correct Answer : C

Answer Justification :

prelims.insightsonindia.com 37
Online Prelims TEST - 25 (TEXTBOOK)
( InsightsIAS Mock Test Series for UPSC Preliminary Exam 2020 )

Justification: SWIFT is a messaging network that financial institutions use to securely transmit
information and instructions through a standardized system of codes. Under SWIFT, each financial
organization has a unique code which is used to send and receive payments.

SWIFT India is a joint venture of top Indian public and private sector banks and SWIFT (Society for
Worldwide Interbank Financial Telecommunication). The company was created to deliver high
quality domestic financial messaging services to the Indian financial community. It has a huge
potential to contribute significantly to the financial community in many domains.

SWIFT does not facilitate funds transfer: rather, it sends payment orders, which must be settled by
correspondent accounts that the institutions have with each other. The SWIFT is a secure financial
message carrier — in other words, it transports messages from one bank to its intended bank
recipient. Its core role is to provide a secure transmission channel so that Bank A knows that its
message to Bank B goes to Bank B and no one else. Bank B, in turn, knows that Bank A, and no one
other than Bank A, sent, read or altered the message en route. Banks, of course, need to have
checks in place before actually sending messages.

Q Source: April 2019 current affairs

51 Consider the following statements.


1. India established the statutory Protection of Plant Varieties and Farmers Rights (PPV&FR)
Authority in compliance to the TRIPS Agreement.
2. “Recognition and protection of the rights of farmers in respect to their contribution in conserving,
improving and making the available plant genetic resources for the development of new plant
varieties” is one of the mandates of PPV&FR.
3. Any University or publicly funded agricultural institution claiming to be breeder of a variety can file
make an application to the PPV&FRA for registration of a variety.

Select the correct answer using the codes below.


A. 1 and 3 only
B. 2 only
C. 2 and 3 only
D. 1, 2 and 3

Your Answer : D
Correct Answer : D

Answer Justification :

Justification: India is signatory of World Trade Organization (WTO). WTO has at least half a dozen
intergovernmental agreements that directly affect agriculture. Under the TRIPS Agreement Article
27(3) (b), which resulted from the negotiations of the Uruguay Round, requires members of protect
plant varieties either by patents or by an effective ‘sui generis’ system of protection or by a
combination of both these systems. In compliance to the TRIPS Agreement India established
Protection of Plant Varieties and Farmers Rights (PPV&FR) Authority, under the Protection of Plant
Varieties and Farmers Rights Act, 2001. PPV & FR Authority has become operational since
th
11 November, 2005. The objectives of the Authority are:

prelims.insightsonindia.com 38
Online Prelims TEST - 25 (TEXTBOOK)
( InsightsIAS Mock Test Series for UPSC Preliminary Exam 2020 )

Establishment of an effective system for protection of plant varieties, the rights of farmers
and plant breeders and to encourage development of new varieties of plants.

Recognition and protection of the rights of farmers in respect to their contribution in


conserving, improving and making the available plant genetic resources for the development
of new plant varieties.

Accelerated agricultural development in the country by stimulation of investment for research


and development both in public and private sector.

Facilitate growth of seed industry to ensure the availability of quality seeds and planting
material to the farmers.

Any of the following persons can make an application to the PPV & FRA for registration of a
variety:-

i) Any person claimed to be a breeder of a variety.


ii) Any person being the assignee of the breeder of a variety.
iii) Any farmer or group of farmers or community of farmers claiming to be the breeder of a
variety.
iv) Any University or publicly funded agricultural institution claiming to be breeder of a variety.

Q Source:
http://seednet.gov.in/Material/IndianSeedSector.htm#Varietal%20Protection%20in%20India

52 Derogations from the Most Favoured Nation (MFN) principle are permitted for forming FTAs under
specific conditions based on the provisions of which of the following WTO Agreements?
1. General Agreement on Tariffs and Trade (GATT)
2. General Agreement on Trade in Services (GATS)

Select the correct answer using the codes below.


A. 1 only
B. 2 only
C. Both 1 and 2
D. None of the above

Your Answer : A
Correct Answer : C

Answer Justification :

Justification: Article 1 of GATT (General Agreement on Tariffs and Trade) which enunciates the
most favoured nation (MFN) principle of WTO states that "any advantage, favour, privilege, or
immunity granted by any contracting party to any product originating in or destined for any other
country shall be accorded immediately and unconditionally to the like product originating in or

prelims.insightsonindia.com 39
Online Prelims TEST - 25 (TEXTBOOK)
( InsightsIAS Mock Test Series for UPSC Preliminary Exam 2020 )

destined for the territories of all other contracting parties."

However, derogations from this MFN principle are permitted for forming FTAs under specific
conditions as per the following provisions of the WTO Agreements:

Article V of GATS (General Agreement on Trade in Services) for services.

The specific conditions under Article XXIV of the GATT permitting FTAs, are: FTA members shall
not erect higher or more restrictive tariff or non-tariff barriers on trade with non-members than
existed prior to the formation of the FTA.

Elimination of tariffs and other trade restrictions be applied to "substantially all the trade between
the constituent territories in products originating in such territories." Elimination of duties and
other trade restrictions on trade within the FTA to be accomplished "within a reasonable length of
time," meaning a period of no longer than 10 years

Q Source: Ministry of Commerce and Industry Website

53 Who, among the following, is/are qualified to members of the Finance Commission?
1. Those who have been, or are qualified to be appointed as Judges of a Supreme Court
2. Those who have special knowledge of economics
3. Those who have served in Indian Bureaucracy at senior positions for at least 25 years

Select the correct answer using the codes below.


A. 2 only
B. 1 and 3 only
C. 2 and 3 only
D. 1, 2 and 3

Your Answer : A
Correct Answer : A

Answer Justification :

Justification: The Finance Commission is appointed by the President under Article 280 of the
Constitution. As per the provisions contained in the Finance Commission [Miscellaneous Provisions]
Act, 1951 and The Finance Commission (Salaries & Allowances) Rules, 1951, the Chairman of the
Commission is selected from among persons who have had experience in public affairs, and the four
other members are selected from among persons who: are, or have been, or are qualified to be
appointed as Judges of a High Court; or have special knowledge of the finances and accounts of
Government; or have had wide experience in financial matters and in administration; or have
special knowledge of economics.

Merely being in senior bureaucracy is not a sufficient criteria for appointment. So, S3 is wrong.

Q Source:
https://www.insightsonindia.com/wp-content/uploads/2019/04/InsightsonIndia-Mar-2019-Current-Aff
airs.pdf

prelims.insightsonindia.com 40
Online Prelims TEST - 25 (TEXTBOOK)
( InsightsIAS Mock Test Series for UPSC Preliminary Exam 2020 )

54 Which of these places, frequently seen in news, is closest to the Caspian Sea?

A. Ashgabat
B. Kabul
C. Kandhar
D. Islamabad

Your Answer : A
Correct Answer : A

Answer Justification :

TAPI gas pipeline project is proposed trans-country natural gas pipeline from Turkmenistan to India
through Afghanistan and Pakistan.

The pipe line connects central Asia with south Asia covering 1,840 km.

The pipeline runs through Galkynysh field (Turkmenistan), Afghanistan’s Herat and Kandahar
province, Pakistan’s Multan via Quetta and ends at Fazilka in Punjab (India).

For its security, an inter-government joint security task force (JSTF) has been recommended to
serve as nucleus of safety provided by security consultants.

Q Source: As mentioned above

55 Hayabusa-2 was recently seen in news. Consider the following about it.
1. It’s aim was to destroy potentially dangerous asteroids and sample underground asteroid debris for
study.
2. It is the first spacecraft to return asteroid samples to Earth for analysis.

prelims.insightsonindia.com 41
Online Prelims TEST - 25 (TEXTBOOK)
( InsightsIAS Mock Test Series for UPSC Preliminary Exam 2020 )

Select the correct answer using the codes below.


A. 1 only
B. 2 only
C. Both 1 and 2
D. None of the above

Your Answer : D
Correct Answer : D

Answer Justification :

Justification: Statement 1: Hayabusa2 is an asteroid sample-return mission operated by the


Japanese space agency, JAXA. It was launched in 2014 and rendezvoused with near-Earth asteroid
162173 Ryugu on 27 June 2018. It is in the process of surveying the asteroid for a year and a half,
departing in December 2019, and returning to Earth in December 2020.

It recently dropped an explosive on an asteroid to make a crater. It’s aim was to make a crater on
asteroid. Also, this spacecraft will collect its underground samples to find possible clues to the
origin of the solar system.

Statement 2: Notably, Hayabusa2 is the second Japanese spacecraft to land on an asteroid, after
Hayabusa achieved a similar feat back in 2005.

Hayabusa-1, its predecessor, landed on the asteroid Itokawa in mid-September 2005, and managed
to collect samples in the form of grains of asteroidal material. It returned to Earth with the samples
in June 2010, thereby becoming the first spacecraft to return asteroid samples to Earth for analysis.

Q Source:
https://www.insightsonindia.com/wp-content/uploads/2019/05/InsightsonIndia-Apr-2019-Current-Aff
airs.pdf

56 The objectives of the India Trade Promotion Organisation (ITPO) include:


1. To support and assist small and medium enterprises to access markets both in India and abroad
2. To assists foreign e-commerce companies and aggregators to setup market in India

Select the correct answer using the codes below.


A. 1 only
B. 2 only
C. Both 1 and 2
D. only

Your Answer : A
Correct Answer : A

Answer Justification :

Justification: India Trade Promotion Organisation (ITPO) is the premier trade promotion agency of
India, provides a broad spectrum of services to trade and industry and acts as a catalyst for growth

prelims.insightsonindia.com 42
Online Prelims TEST - 25 (TEXTBOOK)
( InsightsIAS Mock Test Series for UPSC Preliminary Exam 2020 )

of India’s trade. The main Corporate objectives of ITPO are:

To promote external and domestic trade of India in cost effective manner by organizing and
participating in international trade fairs in India and abroad; organizing buyer-seller meets
and contact promotion programmes abroad; conducting overseas market surveys, exchanging
and contact promotion programmes abroad; conducting overseas market surveys, exchanging
and coordinating visits of business delegations, and undertake need based research to
facilitate trade in specific sectors/markets;

To support and assist small and medium enterprises to access markets both in India and
abroad;

To disseminate trade information and facilitate E-commerce/trade (this does not mention
foreign e-commerce organizations, and also this can’t be a preferred policy objective of the
GoI);

To develop quality physical infrastructure, services and management so as to enable holding


of trade promotion events such as conventions and trade exhibitions of international standard;
and

To enlist the involvement and support of the State Governments, other government trade
promotion agencies, trade and industry associations in trade promotion of India’s external and
domestic trade.

Q Source: https://commerce.gov.in/InnerContent.aspx?Id=229

57 Soil colour gives an indication of the various processes going-on in the soil as well as the type of
minerals in the soil. Consider the following with regard to this.
1. Yellow colour is due to hydrated manganese oxides.
2. Black nodules are due to poor drainage.
3. Dark colour is generally due to the accumulation of highly decayed organic matter.
4. Red colour in the soil is due to the abundance of iron oxide under oxidised conditions.

Select the correct answer using the codes below.


A. 1, 3 and 4 only
B. 3 and 4 only
C. 1, 2 and 3 only
D. 2 and 4 only

Your Answer : B
Correct Answer : B

Answer Justification :

prelims.insightsonindia.com 43
Online Prelims TEST - 25 (TEXTBOOK)
( InsightsIAS Mock Test Series for UPSC Preliminary Exam 2020 )

Justification: The red colour in the soil is due to the abundance of iron oxide under oxidised
conditions (well-drainage) in the soil; dark colour is generally due to the accumulation of highly
decayed organic matter; yellow colour is due to hydrated iron oxides and hydroxide; black nodules
are due to manganese oxides; mottling and gleying are associated with poor drainage and/or high
water table.

Abundant pale yellow mottles coupled with very low pH are indicative of possible acid
sulphate soils.

Colours of soil matrix and mottles are indicative of the water and drainage conditions in the
soil and hence suitability of the soil for aquaculture.

Soil colour is described by the parameters called hue, value and chroma. Hue represents the
dominant wave length or colour of the light; value, refers to the lightness of the colour;
chroma, relative purity or strength of the colour.

Q Source: Additional Research: Chapter 9: 7th NCERT Science

http://www.fao.org/docrep/field/003/AC172E/AC172E04.htm

58 Consider the following about the various kinds of ATM operational in the country.
1. White label ATM is owned by the bank but its operation and maintenance if outsourced to a third
party.
2. Brown label ATM is owned and operated by a non-banking entity while cash is provided by the
sponsored bank.
3. Yellow label ATM is used for e-commerce.

Select the correct answer using the codes below.


A. 1 and 2 only
B. 2 and 3 only
C. 3 only
D. None of the above

Your Answer :
Correct Answer : C

Answer Justification :

The description is as below:

prelims.insightsonindia.com 44
Online Prelims TEST - 25 (TEXTBOOK)
( InsightsIAS Mock Test Series for UPSC Preliminary Exam 2020 )

Q Source: April 2019 Current affairs: Insights

59 Plants which are produced in this way are genetically identical to each other and to the parent(s)

A. Autogamy
B. Asexual reproduction
C. Mitosis and meiosis
D. Allogamy

Your Answer : A
Correct Answer : B

Answer Justification :

Justification: Options A, C and are types of sexual reproduction.

Under asexual reproduction, because of the lack of new genetic material, an organism clones itself
through this process and makes genetically identical organisms.

Asexual reproduction can be advantageous and/or disadvatageous. One positive aspect is that it can
create individuals rapidly and in large quantities. Secondly, bypassing the sexual process can help a
plant in times of dryness since motile sperm require water to fertilize the egg. Another advantage
lies in the fact that plants with the desired characteristics can be cloned for economic reasons
(agriculture). However, if something goes wrong, such as as the occurance of a fatal mutation, the
whole society of clones can be terminated. For this reason, farmers are careful in determining how
to propagate their vegetation. In conclusion, the asexual process of reproduction is an important
one to plants.

prelims.insightsonindia.com 45
Online Prelims TEST - 25 (TEXTBOOK)
( InsightsIAS Mock Test Series for UPSC Preliminary Exam 2020 )

th
Q Source: Additional Research: Chapter 12: 7 NCERT Science

60 In the 16th-17th century India, Muqaddam or mandal was the

A. Local police head


B. Adviser to the King
C. Headman of the panchayat
D. Tax collection officer

Your Answer : C
Correct Answer : C

Answer Justification :

Learning: Some sources suggest that the headman was chosen through the consensus of the
village elders, and that this choice had to be ratified by the zamindar.

Headmen held office as long as they enjoyed the confidence of the village elders, failing which they
could be dismissed by them.

The chief function of the headman was to supervise the preparation of village accounts, assisted by
the accountant or patwari of the panchayat.
th
Q Source: Revision: Page 202: Themes in Indian History – II: 12 NCERT

61 Consider the following statements.


1. The SDG goals apply only to developing and Least Developed Countries (LDCs).
2. The Sustainable Development Goals (SDGs) are not legally binding.

Which of the above is/are correct?


A. 1 only
B. 2 only
C. Both 1 and 2
D. None

Your Answer : B
Correct Answer : B

Answer Justification :

Justification: Statement 1: Unlike the MDGs, the SDG framework does not distinguish between
"developed" and "developing" nations. Hence, it is applicable to all countries.

Statement 2: It is not legally binding. Nevertheless, countries are expected to take ownership and
establish a national framework for achieving the 17 Goals.

At the global level, the SDGs and its 169 targets will be monitored and reviewed using a set of

prelims.insightsonindia.com 46
Online Prelims TEST - 25 (TEXTBOOK)
( InsightsIAS Mock Test Series for UPSC Preliminary Exam 2020 )

global indicators.

Q Source: http://www.niti.gov.in/content/frequently-asked-questions

62 The Lieutenant Governor of a Union Territory is empowered to promulgate ordinances only


1. during recess of the assembly.
2. with the prior permission of the President

Which of the above is/are correct?


A. 1 only
B. 2 only
C. Both 1 and 2
D. None

Your Answer : D
Correct Answer : C

Answer Justification :

Justification: Such an ordinance has the same force as an act of the assembly. Every such
ordinance must be approved by the assembly within six weeks from its reassembly.

He can also withdraw an ordinance at any time.

But, he cannot promulgate an ordinance when the assembly is dissolved or suspended. Further, no
such ordinance can be promulgated or withdrawn without the prior permission of the President.

Q Source: Chapter 35: Indian Polity: M Laxmikanth

63 An inverted duty structure for a particular product will tend to discourage its
1. Domestic value addition
2. Associated Foreign Direct Investment
3. Import as finished goods as compared to its raw material

Select the correct answer using the codes below.


A. 1 and 2 only
B. 2 and 3 only
C. 1 and 3 only
D. 2 only

Your Answer : C
Correct Answer : A

Answer Justification :

Concept: Inverted duty structure is a situation where import duty on finished goods is low
compared to the import duty on raw materials that are used in the production of such finished

prelims.insightsonindia.com 47
Online Prelims TEST - 25 (TEXTBOOK)
( InsightsIAS Mock Test Series for UPSC Preliminary Exam 2020 )

goods.

For example, suppose the tariff (import tax) on the import of tyres is 10% and the tariff on the
imports of natural rubber which is used in the production of tyres is 20%; this is a case of inverted
duty structure.

Justification: Statement 1: When the import duty on raw materials is high, it will be more difficult
to produce the concerned good domestically at a competitive price. Several industries depend on
imported raw materials and components. High tax on the raw materials compels them to raise price.

The disadvantage of the inverted duty structure increases with the increased use of imported
raw materials. An inverted duty structure discourages domestic value addition.

Statement 2: On the other hand, foreign finished goods will be coming at a reduced price because of
low tax advantage. In conclusion, manufactured goods by the domestic industry becomes
uncompetitive against imported finished goods.

In such a case, even foreign investors would not be interested in setting up a firm for
production in the country.

Statement 3: It will be just the opposite.

Q Source:
http://www.insightsonindia.com/2018/02/07/insights-daily-current-affairs-07-february-2018/

64 Micronesia is

A. The lower end of the Arakan Mountains in South-east Asia


B. An extension of the West Asian Peninsular plateau
C. A sub-region of Oceania in the western Pacific Ocean
D. The Northern region of Russia extending into the Arctic Ocean

Your Answer : C
Correct Answer : C

Answer Justification :

Learning: It is composed of thousands of small islands in the western Pacific Ocean. It has a shared
cultural history with two other island regions, Polynesia to the east and Melanesia to the south.

The region has a tropical marine climate, and is part of the Oceania ecozone. There are four main
archipelagos along with numerous outlying islands.

The Micronesia region encompasses five sovereign, independent nations—the Federated States of
Micronesia, Palau, Kiribati, Marshall Islands, and Nauru—as well as three U.S. territories in the

prelims.insightsonindia.com 48
Online Prelims TEST - 25 (TEXTBOOK)
( InsightsIAS Mock Test Series for UPSC Preliminary Exam 2020 )

northern part: Northern Mariana Islands, Guam, and Wake Island.

Micronesia began to be settled several millennia ago, although there are competing theories about
the origin and arrival of the first settlers.

Q Source: World Map based questions

65 ValuES is a global project that concerns with

A. Assessing ecosystem services


B. Integrating global trade supply networks
C. Promoting the benefits of economic development to LDCs
D. Intergovernmental committee for Ethical Systems

Your Answer : A
Correct Answer : A

Answer Justification :

Justification: Assessing Ecosystem Services – ValuES project aids decision-makers in our partner
countries in recognizing and integrating ecosystem services into policy making, planning and
implementation of specific projects.

It works to:

Analyse existing approaches to ecosystem service assessment and valuation. Identify best
practice case studies and generate lessons learned.

Develop an inventory of methods, tools, and indicators to guide practitioners and decision
makers in the selection of approaches to integrating ecosystem services into different sectoral
and other policies

Provide country-specific advisory services to practitioners and decision makers to identify and
include the value of ecosystem services in specific policies and projects.

Develop and facilitate trainings regarding the selection and application of methods and tools
for the as-sessment and valuation of ecosystem services and how to integrate them into
decision-making processes.

Q Source: http://www.aboutvalues.net/about_values/

66 The Lok Sabha (the House of the People) was first constituted in

A. 1946, when the Cabinet Mission recommended the formation of the Constituent Assembly
prelims.insightsonindia.com 49
Online Prelims TEST - 25 (TEXTBOOK)
( InsightsIAS Mock Test Series for UPSC Preliminary Exam 2020 )

B. 1949, when the constitution was ratified


C. 1950, when the constitution was adopted
D. 1952, after the first general elections

Your Answer : D
Correct Answer : D

Answer Justification :

Learning: The Lok Sabha (House of the People) was duly constituted for the first time on 17 April
1952 after the first General Elections held from 25 October 1951 to 21 February 1952.

The first Session of the First Lok Sabha commenced on 13 May 1952.

The Lok Sabha is composed of representatives of the people chosen by direct election on the basis
of adult suffrage. That is why it is called the popular chamber.

Q Source: http://164.100.47.194/loksabha/FAQ.aspx

67 The United States has again placed India on its ‘Priority Watch List’. What does the listing concern?

A. Tariff barriers to trade


B. Intellectual property rights
C. Ease of doing business
D. Sovereign credit rating

Your Answer : A
Correct Answer : B

Answer Justification :

Justification: “Priority Watch List” and “Watch List” countries are identified by the annual Special
301 Report. “Priority Watchlist countries” are judged by the USTR as having “serious intellectual
property rights deficiencies” that require increased USTR attention

India has been on the priority watch list reportedly for over 25 years, for “lack of sufficient
measurable improvements to its IP framework that have negatively affected US right holders”.

The office of the US Trade Representative identified 11 countries, including India, in its ‘Priority
Watch List’.

The list topped by China also includes Indonesia, Russia, Saudi Arabia and Venezuela. Besides this,
the US Trade body has placed 25 countries, including Pakistan, Turkey and the UAE, on the
watchlist.

Q Source:
https://www.insightsonindia.com/wp-content/uploads/2019/05/InsightsonIndia-Apr-2019-Current-Aff
airs.pdf

prelims.insightsonindia.com 50
Online Prelims TEST - 25 (TEXTBOOK)
( InsightsIAS Mock Test Series for UPSC Preliminary Exam 2020 )

68 With reference to a Biotope, consider the following statements.


1. It is an ecological area that is usually larger than an ecosystem.
2. It is a common practice to isolate biotopes from each other for niche biodiversity propagation.

Which of the above is/are correct?


A. 1 only
B. 2 only
C. Both 1 and 2
D. None

Your Answer : C
Correct Answer : D

Answer Justification :

Justification: Statement 1: It is an ecological area that supports a particular range of biological


communities.

Biotope is almost synonymous with the term habitat.

A biotope is generally not considered to be a large-scale phenomenon. For example, a biotope


might be a neighbouring park, a back garden, even potted plants or a fish tank on a porch.

In other words, the biotope is not a macroscopic but a microscopic approach to preserving the
ecosystem and biological diversity.

Statement 2: It is commonly emphasised that biotopes should not be isolated. Instead biotopes need
to be connected to each other and other surrounding life for without these connections to life-forms
such as animals and plants, biotopes would not effectively work as a place in which diverse
organisms live.

So one of the most effective strategies for regenerating biotopes is to plan a stretch of
biotopes, not just a point where animals and plants come and go. (Such an organic traffic
course is called a corridor.

Q Source: Glossary of important ecological terms

69 Which of the following texts deals with Buddhism?

A. Swayambhu Purana
B. Markandey Purana
C. Bhagavata Purana
D. Skanda Purana

prelims.insightsonindia.com 51
Online Prelims TEST - 25 (TEXTBOOK)
( InsightsIAS Mock Test Series for UPSC Preliminary Exam 2020 )

Your Answer :
Correct Answer : A

Answer Justification :

Learning: Swayambhu Purana is a Buddhist scripture about the origin and development of
Kathmandu valley.

Swayambhu Purana gives details of all the Buddhas who came to Kathmandu. It also provides
information about the first and the second Buddhas in Buddhism.

The Swayambhu Purana is one of the oldest texts of Nepal's Newar Buddhist cult.
th
Q Source: Additional Research: 11 TN History Textbook

70 The President can remove the chairman or any other member of UPSC from the office under the
same procedure and grounds as that of

A. A Judge of Supreme Court


B. Chairman, Rajya Sabha
C. Comptroller and Auditor General of India
D. None of the above

Your Answer : D
Correct Answer : D

Answer Justification :

Justification: The procedure for removal is different in the case of UPSC members and
Chairperson.

President can remove them the office if they are insolvent, engage in paid employment or in
the opinion of the president, unfit to continue in office by reason of infirmity of mind or body.

In addition to these, the president can also remove the chairman or any other member of
UPSC for misbehaviour.

However, in this case, the president has to refer the matter to the Supreme Court for an
enquiry.

If the Supreme Court, after the enquiry, upholds the cause of removal and advises so, the
president can remove the chairman or a member.

Under the provisions of the Constitution, the advise tendered by the Supreme Court in this
regard is binding on the president. During the course of enquiry by the Supreme Court, the
prelims.insightsonindia.com 52
Online Prelims TEST - 25 (TEXTBOOK)
( InsightsIAS Mock Test Series for UPSC Preliminary Exam 2020 )

president can suspend the chairman or the member of UPSC.

Q Source: Chapter 39: Indian Polity: M Laxmikanth

71 Consider the following about Renewable Energy Certificates (RECs), also known as green energy
certificates.
1. Each REC represents the environmental benefits of 1 KWh of renewable energy generation.
2. When you purchase RECs, renewable energy is generated on your behalf.
3. It helps states meet their Renewable Purchase Obligations (RPOs) even if they face certain
constraints owing to their geographical conditions.

Select the correct answer using the codes below.


A. 1, 2 and 3
B. 2 and 3 only
C. 1 and 2 only
D. 3 only

Your Answer : B
Correct Answer : B

Answer Justification :

Justification: Renewable Energy Certificates (RECs), also known as green energy certificates or
tradable renewable certificates are proof that energy has been generated from renewable sources
such as solar or wind power. Each REC represents the environmental benefits of 1MWh of
renewable energy generation. When you purchase RECs, renewable energy is generated on your
behalf.

It is a market based mechanism which will help the states meet their regulatory requirements (such
as Renewable Purchase Obligations (RPOs)) by overcoming the geographical constraints on existing
renewable potential in different states.

RECs unbundle the electricity component (commodity) from the green/environmental attributes of
the power generated from renewable sources. Both the components can then be traded separately.
Thus, RECs help in incentivizing the production of renewable energy over and above the RPO state
limit as tradable certificates are not constrained by the geographical limitations of commodity
electricity.

Learning: RECs are being charged GST, while bundled power (RECs plus electricity, irrespective of
source) or even just electricity are devoid of the same. Renewable Energy (RE) companies have
moved the Delhi High Court, seeking an exemption for Renewable Energy Certificates (RECs) under
the GST.

Q
Source:https://www.insightsonindia.com/wp-content/uploads/2019/05/InsightsonIndia-Apr-2019-Cu
rrent-Affairs.pdf

prelims.insightsonindia.com 53
Online Prelims TEST - 25 (TEXTBOOK)
( InsightsIAS Mock Test Series for UPSC Preliminary Exam 2020 )

72 With reference to certain policy initiatives taken by the Government of India to promote the seed
sector, consider the following:
1. National Food Security Mission and Rashtriya Krishi Vikas Yojna have a bearing on the development
of seeds in India.
2. There is no dedicated National Seeds policy in India.
3. Breeder seed production has been outsourced entirely to private sector agencies.

Select the correct answer using the codes below.


A. 1 and 2 only
B. 2 and 3 only
C. 1 and 3 only
D. 1 only

Your Answer : B
Correct Answer : D

Answer Justification :

Justification: Statement 1: The following policy initiatives have been taken by the Government of
India in seed sector:-

Enactment of the Seeds Act, 1966; Seed Review Team-SRT (1968); National Commission on
Agriculture’s Seed Group (1972); Launching of the World Bank aided National Seeds Programme
(1975-85) in three phases; leading to the creation of State Seeds Corporations, State Seed
Certification Agencies, State Seed Testing Laboratories, Breeder Seed Programmes etc, Seed
Control Order (1983).

Other include: Creation of the Technology Mission on Oilseeds & Pulses (TMOP) in 1986 now
called The Integrated Scheme of Oilseeds, Pulses, Oil Palm and Maize (ISOPOM).

Production and Distribution Subsidy; Distribution of Seed Mini-kits; Seed Transport Subsidy
Scheme (1987); New Policy on Seed Development (1988); Seed Bank Scheme (2000)
National Seeds Policy (2002); The Seeds Bill (2004); Formulation of National Seed Plan (2005);
National Food Security Mission (2007); Rashtriya Krishi Vikas Yojna (2007).

Statement 2:National Seeds Policy, 2002: Thrust Areas

– variety development
– plant variety protection
– seed production
– quality assurance
– seed distribution and marketing
– infrastructure facilities
– transgenic plant varieties
– import of seeds and planting materials
– seed exports
– promotion of domestic private sector seed industry
– strengthening of the monitoring system

prelims.insightsonindia.com 54
Online Prelims TEST - 25 (TEXTBOOK)
( InsightsIAS Mock Test Series for UPSC Preliminary Exam 2020 )

Statement 3: Breeder seed is the progeny of nucleus seed of a variety and is produced by the
originating breeder or by a sponsored breeder. Breeder seed production is the mandate of the
Indian Council of Agricultural Research (ICAR) and is being undertaken with the help of; i) ICAR
Research Institutions, National Research Centres and All India Coordinated Research Project of
different crops; ii) State Agricultural Universities (SAUs) with 14 centres established in different
States; NGOs etc.

Q Source:http://seednet.gov.in/Material/IndianSeedSector.htm

73 In India, who is empowered to declare an area to be a scheduled area, under the constitution of
India?

A. President
B. Governor
C. District Magistrate/Commissioner
D. Chairman, Zila Parishad

Correct Answer : A

Answer Justification :

Learning: The scheduled areas are treated differently from the other areas in the country because
they areinhabited by ‘aboriginals’ who are socially and economically rather backward, and special
efforts need to be made to improve their condition.

Therefore, the whole of the normal administrative machinery operating in a state is not extended to
the scheduled areas and the Central government has somewhat greater responsibility for these
areas’.

The president is empowered to declare an area to be a scheduled area. He can also increase or
decrease its area, alter its boundary lines, rescind such designation or make fresh orders for such
redesignation on an area in consultation with the governor of the state concerned.

Q Source: Chapter 36: Indian Polity: M Laxmikanth

74 What are the atmospheric factors affecting transpiration in plants?


1. Relative humidity
2. Wind and air movement
3. Soil-moisture availability

Select the correct answer using the codes below.


A. 1 and 2 only
B. 2 and 3 only
C. 1 only
D. 1, 2 and 3

prelims.insightsonindia.com 55
Online Prelims TEST - 25 (TEXTBOOK)
( InsightsIAS Mock Test Series for UPSC Preliminary Exam 2020 )

Your Answer : D
Correct Answer : D

Answer Justification :

Concept: Transpiration is the process by which moisture is carried through plants from roots to
small pores on the underside of leaves, where it changes to vapor and is released to the
atmosphere.

Studies have revealed that about 10 percent of the moisture found in the atmosphere is released by
plants through transpiration. The remaining 90 percent is mainly supplied by evaporation from
oceans, seas, and other bodies of water (lakes, rivers, streams).

Justification: The amount of water that plants transpire varies greatly geographically and over
time. There are a number of factors that determine transpiration rates:

Temperature: Transpiration rates go up as the temperature goes up, especially during the growing
season, when the air is warmer due to stronger sunlight and warmer air masses.

Higher temperatures cause the plant cells which control the openings (stoma) where water is
released to the atmosphere to open, whereas colder temperatures cause the openings to close.

Relative humidity: As the relative humidity of the air surrounding the plant rises the transpiration
rate falls. It is easier for water to evaporate into dryer air than into more saturated air.

Wind and air movement: Increased movement of the air around a plant will result in a higher
transpiration rate. This is somewhat related to the relative humidity of the air, in that as water
transpires from a leaf, the water saturates the air surrounding the leaf.

If there is no wind, the air around the leaf may not move very much, raising the humidity of
the air around the leaf. Wind will move the air around, with the result that the more saturated
air close to the leaf is replaced by drier air.

Soil-moisture availability: When moisture is lacking, plants can begin to senesce (premature ageing,
which can result in leaf loss) and transpire less water.

Type of plant: Plants transpire water at different rates. Some plants which grow in arid regions,
such as cacti and succulents, conserve precious water by transpiring less water than other plants.
th
Q Source Additional Research: Chapter 7: 6 NCERT: Science

https://water.usgs.gov/edu/watercycletranspiration.html

75 With reference to National Knowledge Network (NKN), consider the following statements.
1. It aims to connect all universities, research institutions, libraries, laboratories, healthcare and
agricultural institutions across the country.
2. It enables sharing of scientific databases and remote access to advanced research facilities.

Select the correct answer using the codes below.

prelims.insightsonindia.com 56
Online Prelims TEST - 25 (TEXTBOOK)
( InsightsIAS Mock Test Series for UPSC Preliminary Exam 2020 )

A. 1 only
B. 2 only
C. Both 1 and 2
D. None of the above

Correct Answer : C

Answer Justification :

Justification: NKN is a multi-gigabit pan-India network which facilitates the development of India’s
communications infrastructure, stimulates research and creates next generation applications and
services. Aim: With its multi-gigabit capability, NKN aims to connect all universities, research
institutions, libraries, laboratories, healthcare and agricultural institutions across the country to
address such paradigm shift.

It enables collaboration among researchers from different educational networks such as TEIN4,
GARUDA, CERN and Internet2. It also enables sharing of scientific databases and remote access to
advanced research facilities. The leading mission oriented agencies in the fields of nuclear, space
and defence research are also part of NKN.

Role of NKN: Establishing a high-speed backbone connectivity which will enable knowledge and
information sharing amongst NKN connected institutes. Enabling collaborative research,
development and innovation amongst NKN connected institutes. Facilitating advanced distance
education in specialized fields like engineering, science, medicine etc. Facilitating an ultra-high
speed e-governance backbone. Facilitating connection between different sectoral networks in the
field of research

Q
Source:https://www.insightsonindia.com/wp-content/uploads/2019/04/InsightsonIndia-Mar-2019-Cu
rrent-Affairs.pdf

76 Program on Forests (PROFOR) is hosted under the aegis of

A. World Bank
B. UNFCCC Secretariat
C. IUCN Secretariat
D. World Wildlife Fund (WWF)

Your Answer :
Correct Answer : A

Answer Justification :

Justification: The overall objective of program on forests (PROFOR) is to strengthen the capacity
of national forest programs and other forest-related processes to better address poverty alleviation,
sustainable development and forest conservation needs.

prelims.insightsonindia.com 57
Online Prelims TEST - 25 (TEXTBOOK)
( InsightsIAS Mock Test Series for UPSC Preliminary Exam 2020 )

This will be achieved through a strategic partnership between the World Bank, the Food and
Agriculture Organization (FAO), contributing donors, client countries, non-governmental
organizations (NGOs) with special expertise in sustainable forest conservation and development and
innovators in sustainable development in the private sector. Program on Forests (PROFOR) will
generate and disseminate cutting-edge knowledge on critical sectoral and cross-sectoral issues
related to the management, conservation and sustainable development of forest resources.

The Program on Forests (PROFOR) will pursue these themes are: 1) support for analytical research
that will help to improve understanding of strategies that will enhance the role of forest resources
in contributing to poverty alleviation, economic growth and preservation of environmental values.;
2) generation and distribution of knowledge and information on concepts, approaches, strategies,
tools, and best practices related to defined thematic areas to developing countries and to
international organizations; and 3) generation and distribution of lessons learned on the
management of natural family planning (NFP) processes.

Q
Source:http://documents.worldbank.org/curated/en/107541468335688863/Program-on-Forests-PR
OFOR-program-of-work-and-financing

77 Consider the following food sources and the vitamin that they contain:
1. Fish: Vitamin D
2. Nuts and Seeds: Vitamin E
3. Spinach: Vitamin K

Select the correct answer using the codes below.


A. 1, 2 and 3 only
B. 3, 4 and 5 only
C. 1, 2 and 5 only
D. 1, 2, 3, 4 and 5

Your Answer :
Correct Answer : D

Answer Justification :

Justification: Vitamin A: Retinol, retinal, and four carotenoids including beta carotene: Contained
in Liver, orange, ripe yellow fruits, leafy vegetables, carrots, pumpkin, squash, spinach, fish, soy
milk, milk

Vitamin B1: Thiamine: Contained in Pork, oatmeal, brown rice, vegetables, potatoes, liver, eggs

Vitamin D: Fish, eggs, liver, mushrooms

Vitamin D is essential only for people who do not have adequate skin exposure to sunlight, because
the ultraviolet light in sunlight normally promotes synthesis of vitamin D.

While vitamin supplements are important for the treatment of certain health problems, otherwise
healthy people generally receive no benefit from using vitamin supplements.

prelims.insightsonindia.com 58
Online Prelims TEST - 25 (TEXTBOOK)
( InsightsIAS Mock Test Series for UPSC Preliminary Exam 2020 )

Vitamin E : Many fruits and vegetables, nuts and seeds

Vitamin K: Leafy green vegetables such as spinach, egg yolks, liver

Q Source:Based on past year UPSC papers

78 Consider the following statements.


1. The Barabar Caves are the oldest surviving rock-cut caves in India
2. Barabar Caves were dedicated the Jaina sect of India.

Which of the above is/are correct?


A. 1 only
B. 2 only
C. Both 1 and 2
D. None of the above

Your Answer : A
Correct Answer : A

Answer Justification :

Learning: The Barabar Caves are the oldest surviving rock-cut caves in India, mostly dating from
the Maurya Empire (322–185 BCE), some with Ashokan inscriptions.

The caves were used by ascetics from the Ajivika sect, founded by MakkhaliGosala, a contemporary
of Gautama Buddha and Mahavira.

At the site several rock-cut Buddhist and Hindu sculptures are also present.

Barabar Hill contains four caves, namely, Karan Chaupar, Lomas Rishi, Sudama and VisvaZopri.
Sudama and Lomas Rishi Caves are the earliest examples of rock-cut architecture in India.

Q Source: Revision: 12th NCERT: Themes in Indian History – I

79 The fundamental right of “Freedom of speech and expression”has been interpreted by the law courts
to include which of the following rights?
1. Right to know about government activities.
2. Freedom of silence.
3. Right against imposition of pre-censorship on a newspaper.
4. Right to demonstration or picketing but not right to strike.

Select the correct answer using the codes below.


A. 1 and 2 only
B. 1, 2 and 4 only
C. 3 and 4 only
D. 1, 2, 3 and 4

prelims.insightsonindia.com 59
Online Prelims TEST - 25 (TEXTBOOK)
( InsightsIAS Mock Test Series for UPSC Preliminary Exam 2020 )

Your Answer : C
Correct Answer : D

Answer Justification :

Justification: The freedom of speech and expression, in this context, includes:

Right to propagate one’s views as well as views of others.


Freedom of the press.
Freedom of commercial advertisements.
Right against tapping of telephonic conversation.
Right to telecast, that is, government has no monopoly on electronic media.
Right against bundh called by a political party or organisation.
Right to know about government activities.
Freedom of silence.
Right against imposition of pre-censorship on a newspaper.
Right to demonstration or picketing but not right to strike.

Q Source: Chapter 7: Indian Polity: M Laxmikanth

80 Consider the following about the non-core assets of Central Public Sector Enterprises (CPSEs).
1. A non-core asset becomes a recurring lability for a CPSE and needs to be disinvested in for
economic viability.
2. The Department of Investment and Public Asset Management (DIPAM) has issued guidelines for
monetisation of non-core assets of CPSEs.

Select the correct answer using the codes below.


A. 1 only
B. 2 only
C. Both 1 and 2
D. None of the above

Your Answer :
Correct Answer : B

Answer Justification :

Justification: Non-core assets are assets that are either not essential or simply no longer used in a
company’s business operations. Non-core assets are often sold when a company needs to raise cash.
Some businesses sell their non-core assets in order to pay down their debt.

Composition: A non-core asset can be any kind of asset, including an entire subsidiary or a holding
in another company. But often non-core assets are things such as real estate, commodities, natural
resources, currencies or securities. A non-core asset might also be factory or property that is no
longer being used.

Whether an asset is considered noncore is entirely relative to the company. An asset that is non-
core for one company might be core for another.

prelims.insightsonindia.com 60
Online Prelims TEST - 25 (TEXTBOOK)
( InsightsIAS Mock Test Series for UPSC Preliminary Exam 2020 )

The Department of Investment and Public Asset Management (DIPAM) issued guidelines for
monetisation of non-core assets of CPSEs.

Q Source:April 2019 current affairs

81 Bharati is a unified script for nine Indian languages which is being proposed as a common script for
India. It does NOT include which of the following scripts?

A. Gurmukhi
B. Tamil
C. Oriya
D. It includes all the above

Your Answer : D
Correct Answer : D

Answer Justification :

Justification: A team of scientists at IIT Madras have developed a method for reading documents
in Bharati script using a multi-lingual optical character recognition (OCR) scheme.

Bharati is a unified script for nine Indian languages which is being proposed as a common script for
India. It is developed by an IIT Madras team. Bharti script aims to bring down the communication
barriers in India with a common script.

The scripts that have been integrated include Devnagari, Bengali, Gurmukhi, Gujarati, Oriya,
Telugu, Kannada, Malayalam and Tamil. English and Urdu have not been integrated so far.

Q
Source:https://www.insightsonindia.com/wp-content/uploads/2019/05/InsightsonIndia-Apr-2019-Cu
rrent-Affairs.pdf

82 Consider the following about India’s GDP growth rate.


1. Real GDP growth rate has been higher than that of emerging market economies (Average) from
2011-19.
2. India’s share in World GDP has declined constantly since 2011.

Select the correct answer using the codes below.


A. 1 only
B. 2 only
C. Both 1 and 2
D. None of the above

Your Answer : A
Correct Answer : A

Answer Justification :

prelims.insightsonindia.com 61
Online Prelims TEST - 25 (TEXTBOOK)
( InsightsIAS Mock Test Series for UPSC Preliminary Exam 2020 )

Justification: Statement 1:

Statement 2:

Q Source:Volume 2: Economic Survey 2018-19

83 GRAPES-3 experiment, seen in news sometimes back, is designed to study

A. Dark matter

prelims.insightsonindia.com 62
Online Prelims TEST - 25 (TEXTBOOK)
( InsightsIAS Mock Test Series for UPSC Preliminary Exam 2020 )

B. Cosmic rays
C. Infrared radiation in space
D. UV rays

Your Answer : C
Correct Answer : B

Answer Justification :

Justification: The GRAPES-3 experiment (or Gamma Ray Astronomy PeVEnergieS phase-3) located
at Ooty in India started as a collaboration of the Indian Tata Institute of Fundamental Research and
the Japanese Osaka City University, and now also includes the Japanese Nagoya Women’s
University.

GRAPES-3 is designed to study cosmic rays with an array of air shower detectors and a large area
muon detector. It aims to probe acceleration of cosmic rays in the following four astrophysical
settings. These include acceleration of particles to, (i) ~100 MeV in atmospheric electric fields
through muons, (ii) ~10 GeV in the Solar System through muons, (iii) ~1 PeV in our galaxy, (iv)
~100 EeV in the nearby universe through measurement of diffuse gamma ray flux.

For the first time in the world, researchers at the GRAPES-3 muon telescope facility in Ooty have
measured the electrical potential, size and height of a thundercloud that passed overhead on
December 1, 2014.

Q Source:https://www.insightsonindia.com/2019/03/25/grapes-3/

84 Toluene, that was recently flagged as an environmental hazard, is released from

A. Petrochemical factories
B. Brick kilns
C. Fertilizer processing
D. Cheese factories

Your Answer : A
Correct Answer : A

Answer Justification :

Justification: Toluene is a colourless clear aromatic hydrocarbon also referred as methyl benzene.

Toluene is found in crude oil and is predominantly used as an appreciable solvent. It is used to make
aviation fuel in addition to being used in adhesives, dyes, finger nail polishes etc.

Toluene is one of the petrochemical wastes that get released without treatment from industries
such as refineries, paint, textile, paper and rubber.

Toluene has been reported to cause serious health problems to aquatic life, and studies point that it
has genotoxic and carcinogenic effects on human beings.

prelims.insightsonindia.com 63
Online Prelims TEST - 25 (TEXTBOOK)
( InsightsIAS Mock Test Series for UPSC Preliminary Exam 2020 )

Q Source:In news sometime back

85 Which of the following authorities, in the capacity of holding that office as the primary duty, is NOT
appointed by the President of India?

A. Chairman, UPSC
B. Chief Election Commissioner of India
C. Chairman, NITI Aayog
D. All of the above are appointed by the President.

Your Answer : C
Correct Answer : C

Answer Justification :

Justification:Option C: Prime Minister is the ex-officio chairman of NITI Aayog. So, C is incorrect.

The NITI Aayog comprises the following:

The Prime Minister as the Chairperson.


A Governing Council composed of Chief Ministers of all the States and Union territories with
Legislatures and lieutenant governors of Union Territories(except Delhi and Pondicherry)
Regional Councils composed of Chief Ministers of States and Lt. Governors of Union
Territories in the region to address specific issues and contingencies impacting more than one
state or a region.
Full-time organizational framework composed of a Vice-Chairperson, five full-time members,
and some expert members

Q Source: Chapter 36: Indian Polity: M Laxmikanth

86 Appliances which need to have a BEE energy rating label mandatorily include
1. Electric Geyser
2. Tubular Fluorescent Lamps
3. Oven

Select the correct answer using the codes below


A. 1 only
B. 2 and 3 only
C. 3 only
D. 1 and 2 only

Your Answer : D
Correct Answer : D

Answer Justification :

Justification: One of the most common myths about the BEE Star Rating is that it is solely based

prelims.insightsonindia.com 64
Online Prelims TEST - 25 (TEXTBOOK)
( InsightsIAS Mock Test Series for UPSC Preliminary Exam 2020 )

on the appliance’s power consumption. A lot of factors contribute into formulating the final star
rating on the product.

Although the manufacturers rate and label the product, BEE sets all the standards and norms which
need to be followed while rating an appliance.

While some category of appliances necessarily have to be given a star rating, for others it’s optional.
Appliances which need to have an energy rating label mandatorily: Frost-free refrigerator, Tubular
Fluorescent Lamps, Room Air-Conditioners, Distribution Transformer, Colour TV, CST AC, Direct
Cool Refrigerator and Electric Geyser. The appliances with the lowest energy consumption in a
product category are given the most stars and those with the highest energy consumption are given
the least.

The Ministry of Power has announced that two more electrical appliances microwave ovens and
washing machines will now be assigned star ratings based on their energy efficiency metrics. The
programme of star rating of Microwave Ovens and Washing Machines will be implemented on a
voluntary basis and will be valid up to December 31, 2020.

Q Source:Based on past year UPSC papers

87 A State can be designated into a Union Territory (UT) through

A. by an executive order of the President


B. by a law enacted by the Parliament
C. by a constitutional amendment made by Parliament and agreed by the President
D. by a resolution passed by the Legislatures of the concerned states and assented to by the
President

Your Answer : B
Correct Answer : B

Answer Justification :

Learning: A2 of the Constitution says that Parliament may by law admit into the Union, or
establish, new States on such terms and conditions, as it thinks fit.

The power conferred on Parliament by clause (a) includes the power to form a new State or Union
territory by uniting a part of any State or Union territory to any other State or Union territory

However, such a Bill shall be introduced in either House of Parliament only on the recommendation
of the President and then sent to the concerned state legislatures for their views. These views or
recommendations are not binding though.

Q Source: Chapter 3: Indian Polity: M Laxmikanth

88 Roc, that was recently seen in news


1. is the World’s smallest aircraft capable of commercial operation
2. can be used to drop rockets and space vehicles upto a certain weight

prelims.insightsonindia.com 65
Online Prelims TEST - 25 (TEXTBOOK)
( InsightsIAS Mock Test Series for UPSC Preliminary Exam 2020 )

Select the correct answer using the codes below.


A. 1 only
B. 2 only
C. Both 1 and 2
D. None of the above

Your Answer :
Correct Answer : B

Answer Justification :

Justification: The world’s largest aircraft, Roc, recently took off over the Mojave Desert in
California. It is carbon-composite plane built by Stratolaunch Systems Corp

It has a wingspan the length of an American football field and is powered by six engines on a twin
fuselage. The plane is designed to drop rockets and other space vehicles weighing up to 500,000
pounds at an altitude of 35,000 feet and has been billed by the company as making satellite
deployment as “easy as booking an airline flight.”

Q
Source:https://www.insightsonindia.com/wp-content/uploads/2019/05/InsightsonIndia-Apr-2019-Cu
rrent-Affairs.pdf

89 Household Consumption Expenditure Survey is conducted by

A. National Sample Survey Organization (NSSO)


B. Economic Adviser, Government of India
C. Central Statistical Commission (CSC)
D. Indian Statistics Institute (ISI)

Your Answer : A
Correct Answer : A

Answer Justification :

Justification: The following excerpts from Economic survey is useful:

prelims.insightsonindia.com 66
Online Prelims TEST - 25 (TEXTBOOK)
( InsightsIAS Mock Test Series for UPSC Preliminary Exam 2020 )

Q Source:Volume 2: Economic Survey 2018-19

90 Why are weeds unwanted in crop farms?


1. They serve as hosts for crop diseases or provide shelter for insects to overwinter.
2. They compete with the crop for nutrients and water.
3. They reduce crop quality by contaminating the commodity.

prelims.insightsonindia.com 67
Online Prelims TEST - 25 (TEXTBOOK)
( InsightsIAS Mock Test Series for UPSC Preliminary Exam 2020 )

Select the correct answer using the codes below.


A. 1 and 2 only
B. 2 and 3 only
C. 1 and 3 only
D. 1, 2 and 3

Your Answer : D
Correct Answer : D

Answer Justification :

Justification: Weeds posses one or more of the following:

abundant seed production;


rapid population establishment;
seed dormancy;

Weeds are troublesome in many ways. Primarily, they reduce crop yield by competing for water,
light, soil nutrients, and space. Other problems associated with weeds in agriculture include:

reduced crop quality by contaminating the commodity;


interference with harvest;
serve as hosts for crop diseases or provide shelter for insects to overwinter;
limit the choice of crop rotation sequences and cultural practices; and
production of chemical substances which are toxic to crop plants (allelopathy), animals, or
humans.
th
Q Source Additional Research: Chapter 7: 6 NCERT: Science

https://extension.psu.edu/introduction-to-weeds-what-are-weeds-and-why-do-we-care

91 Red sanders (Pterocarpus santalinus), known for its rich hue and therapeutic properties, is high in
demand across Asia. Consider the following about it.
1. It is endemic to India.
2. Its export is banned in India given its endangered classification by IUCN.

Select the correct answer using the codes below.


A. 1 only
B. 2 only
C. Both 1 and 2
D. None of the above

Your Answer : C
Correct Answer : A

Answer Justification :

Justification: Red Sanders is popular for use in cosmetics and medicinal products as well as for

prelims.insightsonindia.com 68
Online Prelims TEST - 25 (TEXTBOOK)
( InsightsIAS Mock Test Series for UPSC Preliminary Exam 2020 )

making furniture, woodcraft and musical instruments.

Its popularity can be gauged from the fact that a tonne of red sanders costs anything between Rs 50
lakh to Rs 1 crore in the international market.

The tree is endemic to several districts in Andhra Pradesh and some parts of Tamil Nadu and
Karnataka. But overexploitation prompted the Union government in the 1980s to recommend
inclusion of red sanders in Appendix II of CITES, which says “trade must be controlled in order to
avoid utilisation incompatible with their survival”. The species was listed in Appendix II of CITES in
1995, and subsequently export of red sanders was prohibited in 2004.

In 2010, when the CITES was planning to suspend trade of red sanders obtained from India, the
government submitted a Non-Detriment Finding (NDF) report saying it must be allowed to export
from cultivated sources. So in 2012, India got an export quota on red sanders from CITES, under
which the country could export 310 tonnes of red sanders obtained from “artificially propagated”
sources (grown on farms) and 11,806 tonnes of wood from seized sources

Statement 2:The Directorate General of Foreign Trade (DGFT), an agency of the Ministry of
Commerce and Industry, has revised its export policy to permit the export of red sanders if it is
obtained from cultivated land.

Q Source:April 2019 current affairs

92 With reference to some of the events 1946 onwards in Modern India, consider the following:
1. The Muslim League refused to participate in the deliberations of the Constituent Assembly and
pressed for the separate state for Pakistan.
2. A Cabinet Mission was sent that proposed convening of a Constituent Assembly comprising
members elected by the provincial legislatures and nominees of the Indian states.
3. An interim Government was formed headed by Jawaharlal Nehru.

Select the correct answer using the codes below.


A. 1 only
B. 2 and 3 only
C. 1 and 2 only
D. 1, 2 and 3

Your Answer : B
Correct Answer : D

Answer Justification :

Justification: At the conclusion of the Second World War, the Labour Party, under Prime Minister
Clement Richard Attlee, came to power in Britain. The Labour Party was largely sympathetic
towards Indian people for freedom.

A Cabinet Mission was sent to India in March 1946, which after a careful study of the Indian
political scenario, proposed the formation of an interim Government and convening of a Constituent
Assembly comprising members elected by the provincial legislatures and nominees of the Indian

prelims.insightsonindia.com 69
Online Prelims TEST - 25 (TEXTBOOK)
( InsightsIAS Mock Test Series for UPSC Preliminary Exam 2020 )

states.

An interim Government was formed headed by Jawaharlal Nehru. However, the Muslim League
refused to participate in the deliberations of the Constituent Assembly and pressed for the separate
state for Pakistan.

Lord Mountbatten, the Viceroy of India, presented a plan for the division of India into India and
Pakistan, and the Indian leaders had no choice but to accept the division, as the Muslim League was
adamant.

Q Source: Revision: Freedom movement

93 In terms of longitudinal position, the city that lies farthest from the Indian Meridian is

A. Chennai
B. Lucknow
C. Kanpur
D. Bengaluru

Your Answer : A
Correct Answer : D

Answer Justification :

Learning:Rest of them nearly align with the Indian meridian.

prelims.insightsonindia.com 70
Online Prelims TEST - 25 (TEXTBOOK)
( InsightsIAS Mock Test Series for UPSC Preliminary Exam 2020 )

Q Source: Map based questions: India

94 Capital receipts of the government do NOT include

A. Borrowings of the government from the public


B. Grants received from international organisations
C. Tariff duties received on imports
D. Both (b) and (c)

Your Answer : D
Correct Answer : D

Answer Justification :

Learning:Option B and C:Both formpart of the revenue receipt.

The main items of capital receipts are loans raised by the government from the public which are
called market borrowings, borrowing by the government from the Reserve Bank and commercial
banks and other financial institutions through the sale of treasury bills, loans received from foreign
governments and international organisations, and recoveries of loans granted by the central
government.

Other items include small savings (Post-Office Savings Accounts, National Savings Certificates, etc),

prelims.insightsonindia.com 71
Online Prelims TEST - 25 (TEXTBOOK)
( InsightsIAS Mock Test Series for UPSC Preliminary Exam 2020 )

provident funds and net receipts obtained from the sale of shares in Public Sector Undertakings
(PSUs).
th
Q Source: Chapter 5: 12 Macroeconomics NCERT

95 Which of the following acts abolished the office of the secretary of state for India as well that of
Viceroy?

A. Government of India Act 1935


B. Independence Act, 1947
C. Act of 1919
D. Councils Act of 1892

Your Answer : B
Correct Answer : B

Answer Justification :

Learning: The Act abolished the office of viceroy and provided, for each dominion, a governor-
general, who was to be appointed by the British King on the advice of the dominion cabinet.

His Majesty’s Government in Britain was to have no responsibility with respect to the Government
of India or Pakistan.

It also abolished the office of the secretary of state for India and transferred his functions to the
secretary of state for Commonwealth Affairs.

Q Source: Chapter 1: Indian Polity: M Laxmikanth

96 Turing Award is given in which of the following fields?

A. Mathematics
B. Computing
C. Philosophy
D. Telecommunications

Your Answer : A
Correct Answer : B

Answer Justification :

Justification: Three pioneers in artificial intelligence — a senior Google executive, Facebook’s


chief AI scientist, and an academic — were announced as the winners for this year’s A M Turing
Award.

Turing awardis often described as the “Nobel Prize for computing”. Given by the Association for
Computing Machinery (ACM), the A M Turing Award carries $1 million as prize money. It is for

prelims.insightsonindia.com 72
Online Prelims TEST - 25 (TEXTBOOK)
( InsightsIAS Mock Test Series for UPSC Preliminary Exam 2020 )

major contributions of lasting importance to computing. First awarded in 1966, it has been awarded
annually for 53 years so far to 70 recipients.

The award is named after British mathematician and computer scientist Alan Mathison Turing,
whose work in codebreaking is credited with having played a decisive role in World War II. He led a
British team that worked out a way to decrypt intercepted messages, which had been encrypted on
Enigma machines developed by the Germans. Apart from the award, the Turing machine, used in
computing, is named after the pioneer who is considered the father of theoretical computer science
and artificial intelligence.

Q Source:As mentioned above

97 Consider the following statements.


1. Muons and other particles are produced when cosmic rays bombard air particles surrounding the
earth.
2. Muons can have either only positive or only neutral charge.

Select the correct answer using the codes below.


A. 1 only
B. 2 only
C. Both 1 and 2
D. None of the above

Your Answer :
Correct Answer : A

Answer Justification :

Justification: Muons and other particles are produced when cosmic rays bombard air particles
surrounding the earth.

The muons produced can have positive or negative charge. When a positively charged muon falls
through a cloud, it loses energy.

If its energy falls below 1 giga electron volt (GeV), which is the threshold of detection of the
GRAPES-3 muon telescope, it goes undetected. On the contrary, a negatively charged muon gains
energy when falling through the cloud and gets detected.

Since there are more positive than negative muons produced in nature, the two effects don’t cancel
out, and a net change in intensity is detected.

Q Source:https://www.insightsonindia.com/2019/03/25/grapes-3/

98 Consider the following statements about the August Offer of 1940 of the British government.
1. It suggested that a representative “Constitution Making Body” shall be appointed immediately after
the war.
2. It turned down the demand of the Indian National Congress (INC) to set up a national Government

prelims.insightsonindia.com 73
Online Prelims TEST - 25 (TEXTBOOK)
( InsightsIAS Mock Test Series for UPSC Preliminary Exam 2020 )

at the centre.

Select the correct answer using the codes below.


A. 1 only
B. 2 only
C. Both 1 and 2
D. None of the above

Your Answer : C
Correct Answer : A

Answer Justification :

Justification: After the Congress ministries (in the Provinces) resigned in 1939, the British asked
again for the support of the Congress in WW-II.

Later in 1940, INC passed a resolution offering the British Government support in war, if a
provisional National Government is setup at Centre.

This was responded by Lord Linlithgow in the sort of a proposal which is called August Offer.

The august Offer turned down the demand of the Congress to set up a national Government at the
centre but proposed the following:

A representative “Constitution Making Body” shall be appointed immediately after the war.
The number of the Indians in the Viceroy’s Executive council will be increased.
A war advisory Council would be set up.
The Congress did not approve the August Offer.

Q Source:Important events: INM

99 Consider the following statements.


The statutory grants under Article 275 (both general and specific)
1. are given to the states on the recommendation of the Finance Commission.
2. provides for specific grants for promoting the welfare of the scheduled tribes in a state or for
raising the level of administration of the scheduled areas

Select the correct answer using the codes below.


A. 1 only
B. 2 only
C. Both 1 and 2
D. None of the above

Your Answer :
Correct Answer : C

Answer Justification :

prelims.insightsonindia.com 74
Online Prelims TEST - 25 (TEXTBOOK)
( InsightsIAS Mock Test Series for UPSC Preliminary Exam 2020 )

Justification: A275 empowers the Parliament to make such grants to the states which are in need
of financial assistance and not to every state. Also, different sums may be fixed for different states.
These sums are charged on the Consolidated Fund of India every year.

Apart from this general provision, the Constitution also provides for specific grants for promoting
the welfare of the scheduled tribes in a state or for raising the level of administration of the
scheduled areas in a state including the State of Assam.

The statutory grants under Article 275 (both general and specific) are given to the states on the
recommendation of the Finance Commission.

Q Source: Chapter 14: Indian Polity: M Laxmikanth

100 In India, E-way Bill is needed when

A. Moving cargo over long distances


B. Entering an Inner Line Permit (ILP) state
C. Crossing to High Seas from territorial waters
D. Navigating a protected area

Your Answer : A
Correct Answer : A

Answer Justification :

Learning: E-way is an electronic billing system for traders who are moving their goods for sale
beyond 10 km radius.

E-way is valid for all-India movement and transporters can generate the bill electronically and
in self-service mode. Under the system, there is no need to visit any tax office and check post.
While the mandatory compliance of inter-state E-way bill may roll-out soon once again,
however, for intra-state, the date is June 1 2018.
States such as Kerala, Rajasthan, Uttarakhand and Karnataka have already implemented the
e-way bill system.
However, some traders are not happy with the move. Traders are demanding simplification of
the process for generation of these electronic receipts. Explaining that it will be more of a
hindrance than compliance, traders say the move will force them to shut shops.

Q Source:Based on past year UPSC papers

prelims.insightsonindia.com 75

You might also like